Anda di halaman 1dari 92

Abdominal Pain

A 17-year-old boy is taken to the emergency department because he has developed severe abdominal pain. The pain began abruptly several hours previously, and was felt initially in the periumbilical region, but later shifted to the right lower quadrant. The boy had initially felt somewhat nauseous, but this has passed. On physical examination, he is noted to have localized pain on cough and to be running a low-grade fever.
Q1

Examination of the abdomen demonstrates right lower quadrant tenderness at the junction of the middle and outer thirds of the line joining the umbilicus to the anterior superior spine of the iliac. This location is known as which of the following?
/ / / / /

A. Gubernaculum B. Langer's line C. Linea alba D. McBurney's point E. Tunica albuginea

Q2

Which of the following is the most likely diagnosis?


/ / / / /

A. Appendicitis B. Diverticulitis C. Gallstones D. Rectal ulcer E. Renal colic

Q3

The patient also exhibits an increase in pain in the right lower quadrant from the passive extension of the right hip joint. This finding suggests that the inflammation also involves which of the following?
/ / / / /

A. BIadder B. External oblique muscle C. Femur D. IIiopsoas muscle E. Transverse abdominal muscle

Q4

The patient is prepared for immediate surgery. Cefotaxime is administered before, during, and after surgery. The specimen, once removed, is sent to the laboratory for pathology and bacteriologic culture. A malodorous pus surrounds the serosa of the surgical specimen, and a mixed gram-negative flora is cultured. Rapid enzyme tests for beta-Iactamase production are positive. Which of the following drugs should be added to the initial cefotaxime regimen?
/ / / / /

A. Bacitracin B. CIavulanic acid C. CIindamycin D. Isoniazid E. Vancomycin

Q5

The patient's postoperative recovery is uneventfuI, but 10 days after discharge, he returns to his physician complaining of continuous low-grade fever. An abscess is drained transrectally, and organisms are cultured from the pus. Which of the following is an attribute of this organism that makes it an important abscess former?
/ / / / /

A. It is an anaerobe B. It is an intracellular pathogen C. Its endotoxin lacks 2,3-ketodeoxyoctonate D. Mycolic acid E. Prodigious capsule

Abdominal Pain Case 5 Answers A1


The correct answer is D. The point described is McBurney's point, which overlies the location of the appendix in most individuals. The gubernaculum (choice A) is the fibrous cord that connects the primordial testis or ovary to the anterolateral abdominal wall. Langer's lines (choice B) are the cleavage lines of the skin. The linea alba (choice C) is a sheet-like aponeurosis that covers the anterior abdominal wall.

The tunica albuginea (choice E) is a tough fibrous coat that covers the testis.

A2
The correct answer is A. This patient has a typical presentation for appendicitis, and the diagnosis is confirmed by the presence of localized tenderness at McBurney's point. Diverticulitis (choice B) is usually a disease of middle-aged or older individuals and most commonly affects the left-lower quadrant. Symptomatic gallstone disease (choice C) causes pain and tenderness in the right upper quadrant. Rectal ulcer (choice D) causes pain with stool movement, but does not usually produce tenderness identifiable on abdominal examination.

Renal colic (choice E) usually produces flank or lower back pain. A3


The correct answer is D. This patient has a "positive psoas sign," which is an increase in pain from passive extension of the right hip joint. This maneuver stretches the iliopsoas muscle, which lies behind the appendix and can become secondarily inflamed when the appendiceal inflammation extends through the serosa. The psoas sign is clinically useful in both confirming the appendix as the probable origin of the patient's pain, and indicating that the inflammation is transmural and that the risk of rupture and peritonitis is increased. The bladder (choice A) is located more medially, and is usually not affected by appendicitis. The external oblique (choice B) and transverse abdominal (choice E) muscles are in the anterior and lateral abdominal walls, and do not usually become inflamed with appendicitis.

The femur (choice C) is moved during the extension of the right hip joint, but is not the source of the pain.

A4
The correct answer is B. Clavulanic acid is a beta-lactamase inhibitor, which when administered with beta lactam agents, irreversibly binds and inactivates bacterial betalactamases, thereby permitting the companion drug to disrupt bacterial cell wall synthesis. Suspected appendicitis is usually treated with prompt appendectomy, since delay is associated with increased risk of potentially life-threatening peritonitis and sepsis. Bacitracin (choice A) is not correct, since this drug inhibits bacterial cell wall synthesis by binding to and inhibiting the dephosphorylation of a membrane-bound lipid pyrophosphate. Gram-negative bacteria are resistant to this agent, and it would not have a synergistic effect if administered with a third generation cephalosporin. Clindamycin (choice C) is not correct, because this drug blocks protein elongation by binding to the 50S ribosome. Although it is effective against anaerobic gram-negative bacilli, it would not have a complementary effect when administered with a third generation cephalosporin. Isoniazid (choice D) is not correct because it inhibits the synthesis of mycolic acids for the cell wall of actively dividing Mycobacteria. It would not be effective in the flora of this patient's gut, nor would it act synergistically with third generation cephalosporins.

Vancomycin (choice E) is not correct because it disrupts cell wall synthesis in growing grampositive bacteria. It would not be effective against the flora of this patient's gut, nor would it act synergistically with third generation cephalosporins.

A5
The correct answer is E. Prevotella (Bacteroides) is a frequent cause of abscesses in the intestinal tract because it is a normal flora organism and produces a large capsule, which impedes phagocytosis. Although the genus is anaerobic (choice A), it is not this attribute which causes its formation of abscesses. Prevotella is extracellular, not an intracellular pathogen (choice B). Although Prevotella does indeed have this type of endotoxin (choice C), the absence of this molecule decreases the toxicity of the toxin, and does not contribute to its proclivity toward abscess formation.

Mycobacteria, and not other genera such as Prevotella, are known for their long-chain fatty acids (mycolic acids; choice D).

Abdominal Pain Case 4


A 45-year-old man goes to an emergency department because he is experiencing severe abdominal pain, which is radiating straight through to his back. The pain began several hours after an admitted alcoholic binge, and has not changed in position, although it has become worse.
Q1

Which of the following would be the most likely cause of this type of pain?
/ / / / /

A. Acute appendicitis B. Acute hepatitis C. Acute pancreatitis D. Chronic hepatitis E. Myocardial infarction

Q2

In addition to alcohol use, which of the following is a common predisposing factor for this patient's disease?
/ / / / /

A. Biliary tract stones B. Duodenal cancer C. Gastric carcinoma D. Kidney stones E. Peptic ulcer

Q3

Marked serum elevation of which of the following markers would most strongly substantiate the likely diagnosis?
/ / / / /

A. Acid phosphatase B. Amylase C. Aspartate aminotransferase D. AIkaline phosphatase E. Creatinine kinase

Q4

The patient has a severe course that requires treatment in an ICU. CIinically, he appears similar to patients with sepsis, with fever, elevated white count, hypotension, increased pulse rate, shallow and rapid breathing, oliguria, and a blunted sensorium, in addition to his pain and abdominal tenderness. These clinical findings are most likely related to which of the following?
/ / / / /

A. Activation of the inflammatory cascade B. AIcohol withdrawal symptoms C. AIIergic reaction to alcohol D. Drug toxicity effect E. Secondary infection with mixed flora gut bacteria

Q5

The patient's condition resolves in about two weeks, but he continues to drink after leaving the hospitaI. When seen several years later, he has had a number of similar episodes, and now has chronic severe abdominal pain. CT scan demonstrates a single, smooth-walled, fluid filled space in the tail of the pancreas, which can be reached by the radiologist for CT-guided aspiration with an approach from the back. The fluid aspirated is yellowish, clear, and acellular. Which of the following is the most likely diagnosis?
/ / / / /

A. Pancreatic microcystic adenoma B. Pancreatic mucinous cystadenocarcinoma C. Pancreatic mucinous cystadenoma D. Pancreatic pseudocyst E. Pancreatic solid-cystic tumor

____________________________________________________________________

Abdominal Pain Case 4 Answers A1


The correct answer is C. The typical pain described occurs in approximately 50% of patients with acute pancreatitis. Other patients may have milder pain or even, uncommonly, pain first felt in the lower abdomen. The pain of acute appendicitis (choice A) is often felt first as referred pain near the umbilicus, with tenderness on palpation in the left lower quadrant. Acute hepatitis (choice B) can cause pain referred to the right shoulder. Chronic hepatitis (choice D) does not usually cause pain.

Myocardial infarction (choice E) can cause substernal pain and pain radiating to the left shoulder. A2
The correct answer is A. The overwhelmingly most common predisposing factors for acute pancreatitis are gallstones (more specifically tiny ones that lodge in the extrahepatic bile duct system) and alcohol abuse. Rarely, nearby cancers (choices B and C) can occlude the pancreatic duct system and cause a secondary acute pancreatitis. Kidney stones (choice D) have no relationship with pancreatitis.

Peptic ulcers (choice E) that erode into the pancreas can uncommonly secondarily inflame the pancreas

A3
The correct answer is B. The usual markers for pancreatitis are amylase and lipase. Marked elevation of amylase usually means either pancreatic disease or salivary gland disease; lipase will be elevated in pancreatic disease but not salivary gland disease. If you see elevated amylase on a USMLE question, you should think of pancreatitis or salivary gland disease (mumps, salivary gland stone). However, you should be aware, for your general medical knowledge, that modest elevations of amylase can be seen in a much wider variety of settings (often reflecting either subclinical pancreatic damage or hemoconcentration of pancreatic enzymes), including GI obstruction, mesenteric thrombosis and infarction, macroamylasemia (a genetic condition with abnormal amylase), renal disease, ruptured tubal pregnancy, lung cancer, acute alcohol ingestion, and following abdominal surgery. Associate acid phosphatase (choice A) with diseases involving the prostate and, to lesser degrees, bone, the heart, platelets, and the liver. Associate aspartate aminotransferase (choice C) with diseases of the heart, muscle, liver, pancreas (though not as important for diagnosis as amylase and lipase), and brain. Associate alkaline phosphatase (choice D) with diseases of bone, liver, and to lesser degrees, lung and heart.

Associate creatinine kinase (choice E) with diseases of the heart, muscle, brain, and the general body (trauma, surgery). A4
The correct answer is A. Acute pancreatitis can either be relatively mild, or a severe condition that may cause death. It is thought that, in severe cases, leakage of enzymecontaining pancreatic secretions into the tissues/and or blood stream causes cleavage of precursors, thus strongly activating the complement and inflammatory cascades. These, in turn, produce abundant cytokines, which worsen the symptoms. The clinical result is similar to sepsis, with risk of multi-organ failure and death. The treatment of acute pancreatitis is primarily supportive, and may include careful attention to fluid resuscitation, oxygen supplementation, cardiovascular support, dialysis, management of electrolyte abnormalities, pain control, and total parenteral nutrition.

Alcohol allergy (choice C) or withdrawal (choice B) do not play any additional part in most of these symptoms once the pancreatitis has developed.

Infection (choice E) and drug toxicity (choice D) are also not a necessary part of the clinical picture, although physicians may worry that the patient's general clinical status is masking other, potentially more treatable, problems. A5
The correct answer is D. Pancreatic pseudocyst is a fairly common complication of both acute and chronic pancreatitis, and appears to develop when trapping of pancreatic digestive juices (containing amylase, lipase, and proteases) causes a "digestion" of part of the pancreas, leaving a fluid filled cystic space. The term "pseudocyst", rather than "cyst", is used by purists because the space does not have an epithelial lining, and is hence not a "true cyst". Pseudocysts are usually solitary and typically measure 5-10 cm in diameter. They can be surgically excised (and the surrounding tissue will typically show evidence of chronic pancreatitis in long-standing cases) or sometimes, if the anatomy is favorable, drained into adjacent hollow viscera. Some are medically managed if small.

Most true neoplasms of the pancreas contain (often large numbers of) smaller, multiple, cysts. These tumors can be benign or malignant, and the ones with mucus-secreting epithelium (choices B and C) are more common than those with a serous lining (choices A and E).

Abdominal Pain Case 3


A 64-year-old man with a history of coronary artery disease (CAD) comes to the emergency department with the acute onset of severe, constant, Lower abdominal pain and rectal bleeding. He reports that he previously has had several episodes of similar, but less severe pain. About 12 hours after the onset of pain, the patient began passing copious bright red blood per rectum. He denies nausea, vomiting, sick contacts, or foreign traveI. Initial physical examination reveals a distressed man, who is afebrile, but tachypneic, with scant diffuse abdominal tenderness to palpation. Rectal examination is positive for blood. Laboratory studies reveal a metabolic acidosis with an elevated serum Iactate.

Q1

Which of the following is the most likely diagnosis?


/ / / / /

A. Colon carcinoma B. Infectious colitis C. Inflammatory bowel disease D. Ischemic colitis E. Necrotizing enterocolitis

Q2

The lactate produced from the anaerobic metabolism in the infarcted gut will likely be which of the following?
/ / / / /

A. Exhaled as a fruity odor B. Incorporated into glycogen in the liver C. Incorporated into myoglobin in muscle D. Incorporated into urea in the urine E. Secreted by the kidneys unchanged

Q3

If this patient's disease were drug-induced, which of the following agents would most likely be responsible?
/ / / / /

A. Acetaminophen B. Amiodarone C. Cocaine D. Dexamethasone E. Nitroglycerin

Q4

While the patient is in the emergency department, the pain becomes increasingly severe. Several hours after his initial examination, the patient becomes febrile and is now exquisitely tender to palpation. He writhes in pain when the physician jostles the bed. Air is seen under the diaphragm in an upright chest x-ray film. These new findings suggest which of the following?
/ / / / /

A. Abdominal aortic aneurysm B. Bowel obstruction C. Cholecystitis D. Hypovolemia E. Perforation with peritonitis

Q5

Upon surgical exploration of the abdomen, the colon is dull and dusky from the mid transverse colon to the rectum. The patient has occluded which of the following vessels?
/ / / / /

A. Celiac trunk B. Cystic artery C. External iliac artery D. Inferior mesenteric artery E. Superior mesenteric artery

Abdominal Pain Case 3 Answers


A1 The correct answer is D. A patient with severe abdominal pain and rectal bleeding with an unremarkable physical examination is likely suffering from ischemic colitis. "Pain outof-proportion to examination" is a classic finding for ischemic colitis. The previous episodes of less severe pain represent ischemic angina. An infarction has occurred, as indicated by the rise in serum lactate secondary to the colon's anaerobic metabolism. The history of coronary artery disease also suggests this diagnosis, as the atherosclerotic processes that contribute to his CAD are also likely present in his abdominal vasculature. Colon cancer (choice A) would produce less acute symptoms, but occasionally, colon cancer may present acutely with obstructive symptoms. Patients may have bleeding and abdominal pain, but the pain is typically intermittent and accompanied by nausea, vomiting, abdominal distention, and absence of flatus. Infectious colitis (choice B) is incorrect. While patients may have bleeding and abdominal pain, nothing in the history suggests a disease of infectious origin (no sick contacts or foreign travel). The acute onset also suggests a vascular event, rather than an infectious one.

Inflammatory bowel disease (IBD) (choice C) is incorrect because while the patient reports previous episodes, an elderly man with IBD would likely have a chronic history of abdominal pain and bleeding.

Necrotizing enterocolitis (choice E) affects premature infants and would not be relevant in this setting. A2
The correct answer is B. Lactate is converted into glucose, and then glycogen in the liver by a process know as the Cori cycle. Choice A is incorrect, as lactate would not be exhaled. A fruity odor on the breath would be a sign of ketoacidosis. While some of the carbon from the lactate may be incorporated into peptides via Krebs intermediates (e.g., choice C), the vast majority would be left as carbohydrate. Urea (choice D) represents a means of eliminating nitrogenous waste.

Choice E is wrong, as the kidneys would retain the lactate, rather than excreting it. A3
The correct answer is C. Cocaine is a sympathomimetic drug that indirectly acts on both the alpha and beta adrenergic receptors on the vasculature. As such, cocaine may cause vasospasm in the abdominal vasculature leading to infarction and ischemic colitis. Similar vasospastic events may occur in the coronary vasculature, leading to myocardial infarction. Acetaminophen (choice A) is an analgesic, and would not play a role in producing ischemic colitis. Amiodarone (choice B) is an antiarrhythmic, and would not contribute to ischemic colitis. Dexamethasone (choice D) is a steroidal anti-inflammatory drug. Not only would this medication not cause ischemic colitis, it might mask the symptoms due to its potent antiinflammatory properties.

Nitroglycerin (choice E) is a venodilator, and would not contribute to ischemic colitis. As a venodilator, nitroglycerin is used to treat coronary ischemia by reducing cardiac preload. A4
The correct answer is E. This patient has experienced a bowel perforation. Air under the diaphragm in an upright chest film provides definitive evidence that a hollow viscus has ruptured. Air near the liver on a left lateral decubitus (patient lays with the left side down) is an alternative study to demonstrate perforation. Spillage from the perforated bowel has irritated and inflamed the peritoneum, resulting in peritonitis. Symptoms of peritonitis include extreme, sharp pain exacerbated by jostling (patients often report that the bumpy ride to the emergency department caused extreme pain). Patients will be exquisitely tender to palpation and percussion and may have abdominal rigidity. Fever typically accompanies peritonitis. While an abdominal aortic aneurysm or AAA (choice A) presents as acute abdominal pain, this pain is described as tearing and may radiate to the back. A pulsatile abdominal mass may be palpated. The air on the chest film is also inconsistent with AAA. This patient does not have bowel obstruction (choice B). Signs and symptoms of bowel obstruction include: nausea, vomiting, intermittent abdominal pain, hypovolemia, abdominal distention, absence of flatus, and a "step ladder" bowel pattern on abdominal films. Cholecystitis (choice C) typically presents as right upper quadrant (RUQ) pain, fever, and jaundice. Patients usually have a history of colicky RUQ pain.

While the patient is at risk for hypovolemia (choice D), none of the symptoms listed typify hypovolemia. Signs and symptoms of mild to moderate hypovolemia include malaise, dry mouth, thirst, decreased skin turgor, tachycardia, hypotension, and decreased urine output. A5
The correct answer is D. The inferior mesenteric artery distributes blood to the embryologic hindgut. This includes the distal 1/3 of the transverse colon to the rectum. The rectum is spared because it receives circulation from the inferior rectal artery (not mesenteric).

The celiac trunk (choice A) supplies the embryologic foregut. The first three branches include the splenic artery, the left gastric artery, and the common hepatic artery. This patient has no findings in this distribution. The cystic artery (choice B) supplies the gall bladder. There are no gall bladder findings in this case. The external iliac artery (choice C) gives rise to the vessels of the lower extremity. Symptoms of occlusion or stenosis might include buttock and thigh pain exacerbated by walking. Severe stenosis might give patients buttock and thigh pain, even at rest.

The superior mesenteric artery (choice E) supplies the embryologic hindgut. This extends from the duodenum to the proximal 2/3 of the transverse colon

Abdominal Pain Case 2


A 47-year-old woman presents to the emergency department with cra mping/colicky abdominal pain. The current episode of pain began several hours ago, following a fatty meaI. The pain began slowly, and rose in intensity to a plateau over the course of several hours. The patient reports that she had had several other episodes of similar pain during the past several months, with long intervening periods of freedom from pain. On physical examination, she is noted to have tenderness to deep palpation in the right upper quadrant of the abdomen near the rib cage. The patient also reports that she is experiencing shoulder/back pain at a site she identifies near the right lower scapula, but no tenderness can be elicited during the back and shoulder examination.
Q 1 Which / / / / /

of the following organs is the most likely source of this woman's pain?

A. Appendix B. Diaphragm C. Esophagus D. Gallbladder E. Stomach

Q2

Which of the following techniques would be most appropriate to demonstrate the patient's most likely diagnosis?
/ / / / /

A. Colonoscopy B. CT scan of the abdomen C. Esophagoduodenoscopy D. MRI scan of the abdomen E. UItrasonography

Q3

Following appropriate diagnostic studies, the patient is taken to the surgical suite. During the surgery, the surgeon inserts his fingers from right to left behind the hepatoduodenal ligament. As he does so, his fingers enter which of the following?
/ / / / /

A. Ampulla of Vater B. Common bile duct C. Epiploic foramen D. Greater peritoneal sac E. Portal vein

Q4

During the cholecystectomy, the surgeon ligates the cystic artery. This is typically a branch of which of the following?
/ / / / /

A. Gastroduodenal artery B. Left gastroepiploic artery C. Right gastroepiploic artery D. Right hepatic artery E. Superior pancreaticoduodenal artery

Q5

Pathologic examination of the specimen removed by the surgeon demonstrates the presence of numerous yellow stones (shown above).

These are most likely composed primarily of which of the following?


/ / / / /

A. Bilirubinate B. Calcium phosphate C. Cholesterol D. Cystine E. Struvite

Q6

If this patient had a small stone lodge near the ampulla of Vater, which of the following complications would be most likely to occur?
/ / / / /

A. Crohn disease B. Diabetes mellitus C. Pancreatitis D. Peptic ulcer E. Polyarteritis nodosa

Q7

If this patient had refused surgical treatment, which of the following would be the most appropriate pharmacotherapy to provide definitive treatment and thereby relieve associated pain?
/ / /

A. Ampicillin B. CIofibrate C. Meperidine

/ /

D. Oxycodone E. Ursodiol

Abdominal pain Case 2 Answers A1


The correct answer is D. This woman most likely has gallstones. Cholelithiasis, or the formation of calculi (gallstones) within the gallbladder, is very common in the United States, with over 500,000 cholecystectomies being performed yearly. While many cases of gallstone disease are symptomatic, right upper quadrant pain with referral of the pain to the lower right scapula should specifically suggest gallbladder disease. The pattern of episodes of several hours of pain followed by long periods of freedom from pain is also typical of symptomatic gallstone disease. The appendix (choice A) would most likely cause lower abdominal pain. Pain from irritation of the diaphragm (choice B) can cause right upper quadrant pain and referred pain in the supraclavicular area (rather than the subscapular pain of biliary colic). The absence of right upper quadrant tenderness to palpation, and the history of pain after a fatty meal also argue against this diagnosis. Esophageal pain (choice C) related to regurgitation of gastric contents (heartburn) can occur postprandially, but tends to radiate into the neck, throat, or even face.

Peptic ulcer pain of gastric origin (choice E) is usually described as causing burning, gnawing, or hunger, and may be relieved by eating.

A2
The correct answer is E. Real-time ultrasonography, with 98% sensitivity and 95% specificity, is considered the method of choice for diagnosing possible gallbladder stones. Colonoscopy (choice A) and esophagoduodenoscopy (choice C) might be helpful for excluding alternative diagnoses, but would not themselves establish a diagnosis of gallstone disease.

CT (choice B) and MRI (choice D) scans of the abdomen are expensive tests whose use is not warranted, since real-time ultrasonography performs as well or better. A3
The correct answer is C. The space behind the stomach, hepatoduodenal ligament, and hepatogastric ligament is the omental bursa. This space can be entered by passing through the epiploic foramen of Winslow, as described in the question stem. The common bile duct enters the duodenum through the ampulla of Vater (choice A). The hepatoduodenal ligament contains the common bile duct (choice B), the portal vein (choice E), and the hepatic artery.

The greater peritoneal sac (choice D) lies anterior to the stomach and hepatoduodenal ligament. A4
The correct answer is D. The cystic artery is generally a branch of the right hepatic artery. The gastroduodenal artery (choice A) is a branch of the (common) hepatic artery. The left gastroepiploic artery (choice B) is a branch of the splenic artery. The right gastroepiploic artery (choice C) is a branch of the gastroduodenal artery.

The superior pancreaticoduodenal artery (choice E) is a branch of the gastroduodenal artery.

A5
The correct answer is C. The stones are gallstones, and their yellow color indicates that they are composed of cholesterol. Cholesterol stones are the most common form of gallstones. Risk factors include female sex, multiparity, obesity, increased age (female, fat, forty, and fertile) and North American Indian race. Bilirubinate (choice A) gallstones, which are usually associated with hemolytic anemias, are less common, brown, rather than yellow, and often faceted.

Calcium phosphate (choice B), cystine (choice D), and struvite (choice E) composition can be seen in kidney stones A6
The correct answer is C. A small gallstone obstructing the pancreatic outflow is a well-known cause of acute pancreatitis. The other conditions listed are not caused by gallstones

A7
The correct answer is E. The question is asking, "Which of the following will eradicate a gallstone?" When a gallstone is eliminated the pain will subsequently be eliminated. This question is NOT asking, "which of the following is the most appropriate form of pain control?". Ursodiol (ursodeoxycholic acid) is a hydrophilic bile acid that is used to dissolve small (<> Analgesics and antibiotics, such as ampicillin (choice A), are administered when appropriate, but do not help eradicate the stones. Clofibrate (choice B) is an antihyperlipidemic that is associated with the development of gallstones. High-risk patients, such as diabetics and the elderly, should be watched closely.

As a side note, if this question were asking: "which of the following is the most appropriate form of pain control in this patient", the most appropriate answer would be meperidine. Meperidine (choice C) is the narcotic of choice since it causes the least amount of spasm of the sphincter of Oddi. In other words, meperidine is preferred over oxycodone (choice D).

A 65-year-old man presents to his physician because he has been having increasing difficulty swallowing over the past 2 months. He is still able to swallow liquids, but swallowing solid food now causes severe pain and a sense of fullness behind his sternum. He has lost 18 pounds since his swallowing difficulties began. The patient is referred to a gastroenterologist, who demonstrates a mass lesion of the distal esophagus, which on biopsy is shown to contain cancer. Q1 Which of the following is most important in separating the esophagus from the larynx, and must consequently be carefully passed behind during endoscopy? / A. Arytenoids / B. Cricoid cartilage / C. Epiglottis / D. Pharynx / E. Vocal cords Q2 Which of the following nerves provides the efferent impulses necessary for the esophageal actions that occur during swallowing? / A. Glossopharyngeal / B. Hypoglossal / C. Spinal accessory / D. Trigeminal / E. Vagus Q3 Which of the following approximately represents the proportion of different esophageal cancer types now being observed in the United States? / A. 1/10 adenocarcinoma and 9/10 squamous cell carcinoma / B. 1/3 adenocarcinoma and 2/3 squamous cell carcinoma / C. 1/2 adenocarcinoma and 1/2 squamous cell carcinoma / D. 2/3 adenocarcinoma and 1/3 squamous cell carcinoma / E. 9/10 adenocarcinoma and 1/10 squamous cell carcinoma

Q4 Precancerous metaplasia of the esophageal epithelium gives rise to a mucosa resembling which of the following? / A. Mesothelium / B. Respiratory epithelium / C. Small intestine / D. Squamous epithelium / E. Stomach Q5 Frequent use of which of the following has recently been found to probably have a protective effect against development of esophageal cancer? / A. Acetaminophen / B. Alcohol / C. Aspirin / D. Cigarettes / E. Codeine Q6 Currently, esophageal cancer has which of the following long-term survival rates? / A. Less than 5% / B. 30% / C. 50% / D. 70% / E. More than 95% Dysphagia case 2 answers A1 The correct answer is C. Endoscopists are very careful when guiding the endoscope past the epiglottis, which is a pear-shaped portion of elastic cartilage that can be moved during swallowing to close the larynx, preventing swallowed material from eventually entering the lungs. The arytenoids (choice A) are the site of the attachment of the vocal cords (choice E) within the larynx. The cricoid cartilage (choice B) is in the more distal portion of the larynx. The pharynx (choice D) is shared by the respiratory and gastrointestinal tracts.

A2 The correct answer is E. The vagus nerve supplies the efferent input into the esophagus that is necessary for swallowing. The glossopharyngeal nerve (choice A) provides taste and sensation on the palate, but the only muscle it supplies is the stylopharyngeus. The hypoglossal nerve (choice B) moves the tongue during the initiation of swallowing, but does not innervate the esophagus. The spinal accessory nerve (choice C) plays no role in swallowing. This nerve mediates head and shoulder movement and innervates laryngeal muscles. The trigeminal nerve (choice D) provides general sensation to the mouth and motor innervation to the muscles of mastication. A3 The correct answer is C. More recent statistics indicate that the incidence of adenocarcinoma and squamous cell carcinoma of the esophagus are now roughly equal. Formerly, approximately 2/3 of the esophageal cancers were squamous in origin (choice B). Adenocarcinoma of the esophagus is often found in the distal esophagus. A4 The correct answer is C. This is an indirect question about Barrett's esophagus, which is an important precursor of adenocarcinoma of the esophagus. While Barrett's esophagus was initially defined to be either gastric-type or intestinal-type metaplasia of the esophagus, more recent studies have shown that the actual problem lesion is more likely to be intestinal metaplasia (diagnosed when isolated goblet cells are seen in the epithelium) rather than gastric metaplasia (choice E). Metaplasia to mesothelium (choice A) or ciliated respiratory epithelium (choice B) does not usually occur in the esophagus. The normal epithelium of most of the esophagus is squamous (choice D). A5 The correct answer is C. An interesting new research observation that may be exploited in the future is that the incidence of esophageal cancer appears to be much lower in people who use aspirin frequently. Cigarettes (choice D) and alcohol use (choice B) have been implicated as risk factors for esophageal cancer. Acetaminophen (choice A) and codeine (choice E) have no known effects on the incidence of esophageal cancer.

A6 The correct answer is A. Esophageal cancer is one of the very bad cancers, presently with poor long-term survival. The underlying problem is that the esophagus is only about 3 mm thick, and both metastatic disease and direct spread (often unresectable) to mediastinal structures is common. Active research is presently being undertaken to modify this prognosis by using chemotherapy and radiation therapy prior to surgery, but these modalities have not yet come into widespread use.

A 35-year-old woman consults a physician because she has been having trouble swallowing. She also often experiences chronic heartburn. The physician performs a screening physical examination, and notices that the skin of her hands appears tight and shiny. On specific questioning, she reports having often experienced color changes in her hands from white to blue to red. Q1 Which of the following is the most likely cause of the patient's difficulties with swallowing? / A. Achalasia / B. Adenocarcinoma / C. Chagas disease / D. Scleroderma / E. Squamous carcinoma Q2 The color changes described on the patient's hand are most likely due to which of the following? / A. Arteriolar spasm / B. Blood clots at sites of vascular injury / C. Large artery spasm / D. Platelet clots / E. Stasis blood clots

Q3 Additional findings on physical examination include noting that the skin changes are limited to areas distal to the elbow and knee, the presence of calcified nodules on the extensor surfaces of the forearms, and the presence of telangiectasias on the forearms. This suggests that this patient has which of the following? / A. Bauer syndrome / B. Charcot syndrome / C. CREST syndrome / D. Crigler-Najjar syndrome / E. Dandy-Walker syndrome Q4 If this woman's involved skin were biopsied, which of the following would most likely be seen? / A. Cleft separating the dermis and subcutaneous tissues / B. Epithelial cell hyperplasia / C. Marked dermal fibrosis / D. Narrowing of the basal lamina of small capillaries / E. Thickening of rete pegs Q5 More than 90% of the patients with the limited cutaneous form of this disorder make which of the following autoantibodies? / A. Anti-centromere / B. Anti-DNA topoisomerase l / C. Anti-double-stranded DNA / D. Anti-Golgi / E. Anti-ScI-70 _____________________________________________________________________

Dysphagia case 4 answers A1 The correct answer is D. The tip-off is the reference to the patient's skin changes that are typical for scleroderma, also known as systemic sclerosis. Scleroderma is a disease that may be either predominately limited to the skin or involve many body systems, including the musculoskeletal system, gastrointestinal tract (with esophageal involvement most often symptomatic), cardiorespiratory system, and renal system. Esophageal dysfunction is a common complication of scleroderma. In most patients, the skin changes are obvious, even if the patient has not been previously diagnosed. Rarely, the skin changes may be noticed at an earlier stage, in which the skin of the hands appears puffy and edematous, but not scarred. Achalasia (choice A) and Chagas disease (choice C) are also important causes of dysphagia, but in achalasia, the physical examination is usually normal, and in Chagas disease, you should be able to elicit a history of possible exposure in Central or South America. Reflux esophagitis with risk of progression to Barrett esophagus and adenocarcinoma (choice B) can occur in scleroderma, but it is unlikely, at this early stage in the patient's disease, that she already has adenocarcinoma. The risk of squamous cell carcinoma (choice E) is not increased in this patient. A2 The correct answer is A. The color changes described are typical for Raynaud's phenomenon, which occurs because of changes in perfusion due to arteriolar spasm. Raynaud's phenomenon is common in scleroderma, largely because the subintimal hyperplasia of small vessels characteristic of scleroderma can reduce the luminal diameter by more than 75%. Some authors argue that the vascular changes seen in scleroderma are actually the insult that triggers the subsequent development of fibrosis. Blood clots at sites of vascular injury (choice B) are typical in clots that form in arteries on surfaces such as a fractured atherosclerotic clot. Larger artery spasm (choice C) of vessels damaged by atherosclerosis is thought to contribute to some myocardial infarctions, but is not thought to be an important pathologic mechanism in scleroderma. Platelet clots (choice D) can be seen in some diseases in which platelet function is abnormal, such as essential thrombocytopenia. Stasis blood clots (choice E) commonly are found in venous thromboses.

A3 The correct answer is C. These findings, together with esophageal dysfunction and Raynaud's phenomenon (both of which this patient has), are called the CREST syndrome, also known as limited cutaneous scleroderma. This form of scleroderma has a better longterm prognosis than when the skin changes also involve the trunk (diffuse scleroderma) and more internal organs are additionally involved. Bauer syndrome (choice A) is aortitis and aortic endocarditis as a complication of rheumatoid arthritis. Charcot syndrome (choice B) is intermittent claudication. Crigler-Najjar syndrome (choice D) is a severe familial liver disease. Dandy-Walker syndrome (choice E) is a malformation of the central nervous syndrome. A4 The correct answer is C. In scleroderma, early changes (at the point at which the hands appear swollen, rather than with tight, thick skin) show edema with perivascular infiltrates of CD4+ T cells. At this stage, the collagen fibers are swollen and beginning to degenerate. The smaller vessels may show basal lamina thickening (not narrowing as in choice D) and endothelial (not epithelial cell hyperplasia) cell damage and proliferation. With time, the characteristic marked dermal fibrosis develops, which tends to both narrow (not thicken as in choice E) the rete pegs and attach the dermis tightly (compare with cleft formation as in choice A) to subcutaneous tissues. A5 The correct answer is A. All forms of scleroderma are thought to have a strong autoimmune component, and glucocorticoids and azathioprine are used to suppress the inflammatory complications of scleroderma. (Other drugs that can be used in therapy include penicillamine, which inhibits collagen cross-linking, NSAIDS for pain, and ACE inhibitors to protect the kidney if hypertension or renal damage occurs.) The anticentromere antibody is quite specific for CREST syndrome (96% of cases), and is only seen in a minority of patients with diffuse scleroderma (mainly those with Raynaud's phenomenon) and rarely in systemic lupus erythematosus and mixed connective tissue disease. Anti-DNA topoisomerase I (choice B), also called anti-Scl-70 (choice E) occurs commonly (64-75%) in diffuse scleroderma, but only rarely in CREST syndrome. Anti-double-stranded DNA (choice C) is fairly specific for systemic lupus erythematosus,

although it only occurs in 50-60% of lupus cases. Anti-Golgi antibodies (choice D) are seen most often in systemic lupus erythematosus and Sjgren syndrome.

A 30-year-old man consults a physician because he has been having increasing difficulty swallowing both solids and liquids. Physical examination of the patient is noncontributory. Barium swallow studies show a mostly dilated esophagus with slow passage of barium into the stomach. The very distal part of the esophagus appears narrowed into a "bird's beak." Esophageal manometry shows incomplete relaxation of the lower esophageal sphincter in response to swallowing, high resting lower esophageal pressure, and absent esophageal peristalsis. Q1 The manometry and barium swallow studies most strongly support which of the following diagnoses? / A. Achalasia / B. Adenocarcinoma / C. Barrett esophagus / D. Squamous cell carcinoma / E. Systemic sclerosis

Q2 This patient's condition is most likely due to which of the following? / A. Acid reflux / B. Cancerous destruction / C. Candida infection / D. Fibrosis of the esophageal wall / E. Lack of ganglion cells Q3 Which of the following regulators would most likely inhibit the lower esophageal sphincter

in normal individuals? / A. Acetylcholine and substance P / B. Substance P and nitric oxide / C. Substance P only / D. Vasoactive intestinal polypeptide and acetylcholine / E. Vasoactive intestinal polypeptide and nitric oxide

Q4 Which of the following medications is used to directly relax the lower esophageal sphincter? / A. Diphenoxylate / B. Famotidine / C. Granisetron / D. Isosorbide dinitrate / E. Metoclopramide

Q5 Worldwide, which of the following parasitic diseases is most likely to produce a disorder that clinically resembles this patient's condition? / A. Ascariasis / B. African sleeping sickness / C. Chagas disease / D. Cysticercosis / E. Malaria Dysphagia case 3 answers A1 The correct answer is A. The most likely diagnosis is achalasia. This condition is a neurogenic esophageal disorder that can occur at any age, but frequently is diagnosed when individuals are between the ages of 20 and 40. Characteristically, the swallowing difficulties involve both solid food and liquids. The manometry findings illustrated are typical; the barium swallow findings may be as illustrated or may instead show diffuse esophageal dilation without the "bird's beak" near the lower esophageal sphincter.

Larger cancers of the esophagus (choices B and D) would be more likely to cause either a mass or an ulceration, which would be visible on barium swallow. Very small cancers and Barrett's esophagus (choice C) would require esophagogastroduodenoscopy with biopsy for diagnosis, and would be unlikely to cause dysphagia. Systemic sclerosis (choice E) can involve the esophagus as well, however, physical examination would usually show obvious skin involvement. A2 The correct answer is E. Individuals who have achalasia have been found to have a deficiency of inhibitory ganglion cells within the esophageal wall. This lack causes an imbalance in excitatory and inhibitory neurotransmission, with the result that the lower esophageal sphincter tends to have a higher-than-normal muscle tone and relaxes only with difficulty. Acid reflux (choice A) can cause esophageal irritation, ulceration, and also predisposes for Barrett's metaplasia with subsequent risk of adenocarcinoma of the esophagus. Cancerous destruction (choice B) would produce a mass, or area of stricture or ulceration. Candida infection (choice C) of the esophagus resembles thrush of the mouth, and causes a usually superficial infection. Fibrosis of the esophageal wall (choice D) can be the consequence of ulceration (due to reflux or ingestion of harsh chemicals such as lye) or systemic sclerosis. A3 The correct answer is E. Physiologically important inhibitors of the lower esophageal sphincter include nitric oxide and vasoactive intestinal polypeptide. Physiologically important substances that stimulate the lower esophageal sphincter include acetylcholine and substance P. A4 The correct answer is D. Commonly used medications to relax the lower esophageal sphincter in patients with achalasia include nitrates such as isosorbide dinitrate (remember that nitric oxide physiologically inhibits the lower esophageal sphincter) and calcium channel blockers such as nifedipine (which inhibit calcium flow into the smooth muscle of the lower esophageal sphincter, thereby inhibiting contraction.) For patients in whom medical therapy fails, other options include paralysis of the lower esophageal sphincter with intrasphincteric injection of botulinum toxin, pneumatic dilatation, and a

Heller myotomy (which interrupts the muscles of the lower esophageal sphincter). Diphenoxylate (choice A) is a opiate antidiarrheal, and would not be useful for achalasia. Famotidine (choice B) is an H2 antagonist that would be useful in reducing stomach acidity, but would not directly affect lower esophageal pressure. Other drugs in this class include cimetidine, ranitidine, and nizatidine. Granisetron (choice C) is a 5HT3 antagonist and is used to prevent nausea and vomiting in patients who receive chemotherapy, and after general anesthesia. Other members of this drug class include ondansetron and dolasetron. Metoclopramide (choice E) stimulates gastric motility in patients with gastroparesis and is also a antiemetic agent. A5 The correct answer is C. Chagas disease, which is found in South and Central America and is due to infection with Trypanosoma cruzi, can involve the heart, colon, and esophagus. The esophageal involvement clinically closely resembles achalasia. The adult worms of ascariasis (choice A), or roundworm infection, live principally in the intestine, and can obstruct the intestine or a bile duct; the larvae can migrate to the liver, heart, and lungs. African sleeping sickness (choice B), caused by Trypanosoma brucei and Trypanosoma gambiense, causes lymphadenopathy, rash, and CNS involvement. Cysticercosis (choice D), due to the larval form of the pork tapeworm Taenia solium, can involve subcutaneous tissue, muscle, viscera (but not specifically the esophagus), and, most seriously, the CNS. Malaria (choice E), due to various Plasmodium species, involves the blood, liver, kidney, spleen, and brain, but does not have a specific predilection for the esophagus.

A 30-year-old man consults a physician because he has been having increasing difficulty swallowing both solids and liquids. Physical examination of the patient is noncontributory. Barium swallow studies show a mostly dilated esophagus with slow passage of barium into the stomach. The very distal part of the esophagus appears narrowed into a "bird's beak." Esophageal manometry shows incomplete relaxation of the lower esophageal sphincter in response to swallowing, high resting lower esophageal pressure, and absent esophageal peristalsis.

Q1 The manometry and barium swallow studies most strongly support which of the following diagnoses? / A. Achalasia / B. Adenocarcinoma / C. Barrett esophagus / D. Squamous cell carcinoma / E. Systemic sclerosis Q2 This patient's condition is most likely due to which of the following? / A. Acid reflux / B. Cancerous destruction / C. Candida infection / D. Fibrosis of the esophageal wall / E. Lack of ganglion cells Q3 Which of the following regulators would most likely inhibit the lower esophageal sphincter in normal individuals? / A. Acetylcholine and substance P / B. Substance P and nitric oxide / C. Substance P only / D. Vasoactive intestinal polypeptide and acetylcholine / E. Vasoactive intestinal polypeptide and nitric oxide Q4 Which of the following medications is used to directly relax the lower esophageal sphincter? / A. Diphenoxylate / B. Famotidine / C. Granisetron / D. Isosorbide dinitrate / E. Metoclopramide

Q5 Worldwide, which of the following parasitic diseases is most likely to produce a disorder that clinically resembles this patient's condition? / A. Ascariasis / B. African sleeping sickness / C. Chagas disease / D. Cysticercosis / E. Malaria Dysphagia case 3 answers A1 The correct answer is A. The most likely diagnosis is achalasia. This condition is a neurogenic esophageal disorder that can occur at any age, but frequently is diagnosed when individuals are between the ages of 20 and 40. Characteristically, the swallowing difficulties involve both solid food and liquids. The manometry findings illustrated are typical; the barium swallow findings may be as illustrated or may instead show diffuse esophageal dilation without the "bird's beak" near the lower esophageal sphincter. Larger cancers of the esophagus (choices B and D) would be more likely to cause either a mass or an ulceration, which would be visible on barium swallow. Very small cancers and Barrett's esophagus (choice C) would require esophagogastroduodenoscopy with biopsy for diagnosis, and would be unlikely to cause dysphagia. Systemic sclerosis (choice E) can involve the esophagus as well, however, physical examination would usually show obvious skin involvement. A2 The correct answer is E. Individuals who have achalasia have been found to have a deficiency of inhibitory ganglion cells within the esophageal wall. This lack causes an imbalance in excitatory and inhibitory neurotransmission, with the result that the lower esophageal sphincter tends to have a higher-than-normal muscle tone and relaxes only with difficulty. Acid reflux (choice A) can cause esophageal irritation, ulceration, and also predisposes for Barrett's metaplasia with subsequent risk of adenocarcinoma of the esophagus. Cancerous destruction (choice B) would produce a mass, or area of stricture or ulceration. Candida infection (choice C) of the esophagus resembles thrush of the mouth, and causes a usually superficial infection. Fibrosis of the esophageal wall (choice D) can be the consequence of ulceration (due to reflux or ingestion of harsh chemicals such as lye) or systemic sclerosis.

A3 The correct answer is E. Physiologically important inhibitors of the lower esophageal sphincter include nitric oxide and vasoactive intestinal polypeptide. Physiologically important substances that stimulate the lower esophageal sphincter include acetylcholine and substance P. A4 The correct answer is D. Commonly used medications to relax the lower esophageal sphincter in patients with achalasia include nitrates such as isosorbide dinitrate (remember that nitric oxide physiologically inhibits the lower esophageal sphincter) and calcium channel blockers such as nifedipine (which inhibit calcium flow into the smooth muscle of the lower esophageal sphincter, thereby inhibiting contraction.) For patients in whom medical therapy fails, other options include paralysis of the lower esophageal sphincter with intrasphincteric injection of botulinum toxin, pneumatic dilatation, and a Heller myotomy (which interrupts the muscles of the lower esophageal sphincter). Diphenoxylate (choice A) is a opiate antidiarrheal, and would not be useful for achalasia. Famotidine (choice B) is an H2 antagonist that would be useful in reducing stomach acidity, but would not directly affect lower esophageal pressure. Other drugs in this class include cimetidine, ranitidine, and nizatidine. Granisetron (choice C) is a 5HT3 antagonist and is used to prevent nausea and vomiting in patients who receive chemotherapy, and after general anesthesia. Other members of this drug class include ondansetron and dolasetron. Metoclopramide (choice E) stimulates gastric motility in patients with gastroparesis and is also a antiemetic agent. A5 The correct answer is C. Chagas disease, which is found in South and Central America and is due to infection with Trypanosoma cruzi, can involve the heart, colon, and esophagus. The esophageal involvement clinically closely resembles achalasia. The adult worms of ascariasis (choice A), or roundworm infection, live principally in the intestine, and can obstruct the intestine or a bile duct; the larvae can migrate to the liver, heart, and lungs. African sleeping sickness (choice B), caused by Trypanosoma brucei and Trypanosoma gambiense, causes lymphadenopathy, rash, and CNS involvement. Cysticercosis (choice D), due to the larval form of the pork tapeworm Taenia solium, can involve subcutaneous tissue, muscle, viscera (but not specifically the esophagus), and, most seriously, the CNS. Malaria (choice E), due to various Plasmodium species, involves the blood, liver, kidney, spleen, and brain, but does not have a specific predilection for the esophagus.

TOTAL 43 Questions Abdominal Pain, Actomenaphin, Achlasia, Acid reflux

A 53-year-old man consults a physician because he has begun coughing up sputum tinged with fresh blood. He does not initially report any other symptoms to his physician. When his physician comments on his hoarseness and cough, the patient discounts these symptoms and attributes them to his long smoking history. He cannot say when they began or became worse. Q1 The chronic hoarseness suggests dysfunction of which of the following? / A. Palate / B. Pharynx / C. Tongue / D. Trachea / E. Vocal cords Q2 Laryngoscopy reveal a fungating tumor of the larynx that is located between the false and true vocal cords. This tumor is in which of the following sites? / A. Aryepiglottic fold / B. Infraglottic compartment / C. Piriform recess / D. Supraglottic compartment / E. Ventricle Q3 Biopsy of the mass demonstrates a malignancy. Which of the following is the most likely diagnosis? / A. Adenocarcinoma / B. Lymphoma / C. Oat cell carcinoma / D. Sarcoma / E. Squamous cell carcinoma Q4 In addition to smoking, which of the following is an accepted risk factor for this patient's tumor? / A. Alcohol use

/ B. Cocaine use / C. Coffee use / D. Marijuana use / E. Tea use Q5 The patient returns to clinic three weeks after receiving the news that his cancer is inoperable. His wife reports that he has been more withdrawn, eating and sleeping poorly and "just seems to have lost all hope." On examination, the patient moves very little, never makes eye contact, and admits to "some" depressed mood, Ioss of appetite, and sleep disruption. Which of the following symptoms is of most concern regarding his risk of suicide? / A. Decreased appetite / B. Decreased energy / C. Diminished concentration / D. Guilty and worthless feelings / E. Hopelessness Q6 Some patients present with hoarseness due to a tumor at the lung apex that involves a nerve that is a branch of which of the following? / A. Accessory nerve / B. Glossopharyngeal nerve / C. Hypoglossal nerve / D. Phrenic nerve / E. Vagus nerve ____________________________________________________________________ Hemoptysis Case 3 Answers A1 The correct answer is E. Hoarseness specifically suggests dysfunction of the vocal cords, which produce the sounds that are then articulated to speech with the pharynx (choice B), palate (choice A), tongue (choice C), teeth, and lips. While vocal cord dysfunction is specifically suggested by hoarseness, the anatomic lesion may or may not be present at the level of the vocal cords. Vocal paralysis may be the result of local tumor or trauma, intracranial lesions affecting the nucleus ambiguus or its supranuclear tracts, and lesions

at the base of the skull, neck, or upper portion of the thorax that involve either the vagus nerve or the recurrent laryngeal nerves. Air from the trachea (choice D) passes into the larynx to reach the vocal cords, but tracheal dysfunction does not produce hoarseness. A2 The correct answer is E. The interior of the larynx is divided into 3 compartments: the supraglottic compartment (choice D) above the false vocal cords, the ventricle between the false and true vocal cords, and the infraglottic compartment (choice B) below the true vocal cords. The aryepiglottic fold (choice A) is the upper free border of the quadrangular membrane found in the supraglottic compartment. The piriform recess (choice C) lies behind the thyroid laminae and the lateral wall of the supraglottic compartment. A3 The correct answer is E. The epithelial lining of the larynx is squamous epithelium, and the vast majority of cancers of the larynx are squamous cell carcinomas. Squamous cell carcinoma can present with hoarseness, cough, hemoptysis, or difficulty swallowing. If the symptoms develop insidiously (as in the case), the patient may not seek medical attention until late in the course. While very small laryngeal cancers can be treated successfully with surgery and/or radiation, larger ones are much more problematic, in large part because complete resection of the cancer and any lymph node metastases can be difficult to impossible to perform without compromising the many vital neck structures. Patients treated with partial laryngectomy may retain some speech ability. Patients treated with total laryngectomy can often learn to speak again using esophageal speech (gradual belching of air through the pharyngoesophageal junction), a tracheoesophageal fistula (one way valve between the trachea and the esophagus which makes a sound when air is forced across it), or an electrolarynx (sound source held against the neck). With all three techniques, the sound produced is then turned into articulation by the patient's pharynx, palate, tongue, teeth, and lips. The other tumors listed in the choices are very uncommon in the larynx. A4 The correct answer is A. The only two risk factors that you will need to associate with

laryngeal carcinoma are smoking and alcohol use. Cocaine (choice B), coffee (choice C), and tea (choice E) have not been linked to laryngeal cancer. Marijuana (choice D) has been suggested as a risk factor for oral cancer in a few cases, but a causal association has not been established. A5 The correct answer is E. Hopelessness is an ominous sign and is associated with a higher risk for suicide. Decreased appetite (choice A), decreased energy (choice B), and diminished concentration (choice C) are symptoms indicating the presence of a major depressive episode, or are the result of the medical illness. Guilty and worthless feelings (choice D) are symptoms of depression. They do not predict risk for suicide. A6 The correct answer is E. The larynx has complex innervation. The vocal cords and most of the area of the larynx below them are supplied by the recurrent laryngeal nerve, which is a branch of the vagus nerve that passes beneath the subclavian artery before returning to the neck to innervate the larynx. From above the larynx, the superior laryngeal nerve arises from the vagus and divides into the internal laryngeal nerve and the external laryngeal nerve. The innervation above the vocal cords is by the internal laryngeal branch of the vagus nerve. The cricothyroid muscle (the only muscle of the larynx not supplied by the recurrent laryngeal nerve) is supplied by the external laryngeal branch of the superior laryngeal, which also branches off the vagus, but which contains motor fibers originally derived from the accessory nerve (choice A). The glossopharyngeal nerve (choice B) supplies the pharynx. The hypoglossal nerve (choice C) supplies the tongue. The phrenic nerve (choice D) supplies the diaphragm.

Posted by admin at 8:26 AM 0 comments Labels: adenocarcinoma, Alcohol, Cocaine, Hemoptysis, Lymphoma, malignancy, Marijuana, Sarcoma, Squamous cell carcinoma

Hemoptysis Case 2

A fifty-year-old man presents to his primary care doctor complaining of chronic cough with hemoptysis, weakness, frequent urination, thirst, and a decreased ability to concentrate. Review of systems reveals fever, chills, night sweats, and a twenty-pound weight loss. He has a seventy pack-year smoking history. On examination, his lungs are clear to auscultation, but neurologic examination reveals global hyporeflexia. A chest x-ray film reveals a 4-cm hilar non-cavitary opacity in the left lung. Serum electrolytes show K+ = 2.3 mEq/L. Calcium is within normal limits. Q1 Which of the following is the most likely diagnosis? / A. Adenocarcinoma / B. Bronchioloalveolar carcinoma / C. Mesothelioma / D. Small cell carcinoma / E. Squamous cell carcinoma Q2 Which of the following substances secreted from the lesion is the most likely cause of this patient's electrolyte disturbance? / A. ACTH / B. Aldosterone / C. Angiotensin ll / D. Epinephrine / E. PTH Q3 As the patient's condition progresses, the patient grows increasingly hoarse. Which of the following is the most likely reason for the patient's hoarseness? / A. Enlargement of the mass has severely decreased inspiratory volume. / B. Extension of the mass into the larynx / C. Impingement on cranial nerve XII / D. Impingement on the recurrent laryngeal nerve / E. Metastasis into the speech centers of the brain Q4

This patient is given cisplatin as part of his chemotherapeutic regimen. Which of the following is a known adverse effect of cisplatin? / A. Cardiotoxicity / B. Hemorrhagic cystitis / C. Nephrotoxicity / D. Profound myelosuppression / E. Pulmonary fibrosis Q5 In addition, this patient is given etoposide. Which of the following best describes the mechanism of action of etoposide? / A. Etoposide cross-Iinks DNA / B. Etoposide induces single- and double-stranded breaks in DNA / C. Etoposide inhibits dihydrofolate reductase / D. Etoposide inhibits microtubule formation / E. Etoposide inhibits topoisomerase ll Q6 The patient has been aware of his diagnosis and prognosis for several weeks now. He makes the statement "I had successes and failures but I'm pretty sure l got as much living out of the last 50 years as anybody could." This statement suggests that the patient is experiencing which of the following of Erikson's stages? / A. Ego integrity vs. despair / B. Generativity vs. self absorption / C. Identity vs. role confusion / D. Industry vs. inferiority / E. Intimacy vs. isolation ____________________________________________________________________ Hemoptysis Case 2 Answers A1 The correct answer is D. This patient has classic findings for small cell carcinoma of the lung (also known as oat cell carcinoma). Patients with small cell carcinoma typically present with constitutional symptoms, cough, and hemoptysis. Paraneoplastic syndromes are common with this cancer, and may often be the first symptoms to present. This patient has hypokalemia and many of the symptoms that accompany it: muscular

weakness, frequent urination, thirst, and decreased ability to concentrate. This patient's radiographic findings (non-cavitary hilar mass) also support the diagnosis of small cell carcinoma. Adenocarcinomas (choice A) tend to present as peripheral masses, and paraneoplastic syndromes are rare. Bronchioloalveolar carcinoma (choice B) is a subtype of adenocarcinoma. It arises from the peripheral airways, and while it can present as a discrete mass, it typically is indistinct radiographically as it grows down the airway surfaces. Again, paraneoplastic syndromes are rare. Mesothelioma (choice C) is a pleural malignancy associated with asbestos exposure. They do not present as hilar masses. 50% metastasize, but death usually results from local extension. They tend to produce large effusions that may obscure the mass on chest x-ray films. Squamous cell carcinoma (choice E), like small cell carcinoma, has a strong association with smoking. These tumors may present in a similar fashion, with a more central lung lesion, but paraneoplastic syndromes with squamous cell carcinoma tend to be related to the secretion of a PTH-like substance, resulting in hypercalcemia. Thus the diagnosis of small cell carcinoma is more likely. A2 The correct answer is A. ACTH is often secreted by small cell lung carcinoma. The clinical results are those seen from uninhibited ACTH secretion: the adrenal gland increases the production of glucocorticoids and mineralocorticoids. Thus, patients experience hypokalemia, hyperglycemia, and hypertension. The chronic sequelae of this process (Cushingoid symptoms) are not usually seen, due to the metabolic demands of the neoplasm and the short survival of affected individuals. None of the other hormones listed above are commonly secreted by small cell cancers. A3 The correct answer is D. As seen with many hilar masses, impingement on the recurrent laryngeal nerve produces hoarseness. The recurrent laryngeal nerve courses inferiorly to the aortic arch and then turns superiorly to innervate the intrinsic muscles of the larynx (except the cricothyroid muscle). When this nerve sustains unilateral damage, hoarseness results, as patients have difficulty abducting the vocal cords. Bilateral damage results in acute breathlessness, because both of the vocal cords move to the midline and block the airway.

Enlargement of the mass (choice A) to a size that would affect speech would likely produce pulmonary collapse, not hoarseness. Extension of the mass from the hilum of the lung to the larynx (choice B) would be unlikely. Many critical structures are in this course, and the patient would likely not survive long enough to become hoarse from direct extension. Impingement on cranial nerve XII (choice C) would affect movement of the tongue. Not only would metastasis or direct extension to the anatomic site of this nerve be unlikely, this event would not make a patient hoarse. Metastasis to the speech centers (choice E) would produce more profound speech difficulties. The characteristics of the aphasia would depend upon which speech center is affected ( i.e., fluent vs. expressive aphasia). A4 The correct answer is C. Cisplatin is a heavy metal compound used in the treatment of small cell carcinoma of the lung. Cisplatin cross-links DNA, though it is not a true alkylating agent. The dominant adverse effect seen with its administration is nephrotoxicity, as it is toxic to both proximal and distal renal tubule epithelium. It produces only modest myelosuppression. Doxorubicin causes cardiotoxicity (choice A) and congestive heart failure may result. Cyclophosphamide is a chemotherapeutic agent known to cause hemorrhagic cystitis (choice B). The myelosuppression with cisplatin is modest. Many other chemotherapeutic agents are known to severely suppress the marrow (choice D). Bleomycin is known to cause pulmonary fibrosis (choice E). A5 The correct answer is E. Etoposide acts by inhibiting the enzyme topoisomerase II. Topoisomerases are enzymes that create and repair breaks in DNA during replication. This enzyme is needed to relieve topologic and conformational changes as the DNA is "unzipped" during replication and transcription. Alkylating agents, such as cyclophosphamide, cross-link DNA (choice A). The antitumor antibiotics, such as bleomycin, act by inducing breaks in DNA (choice B). Methotrexate is an antimetabolite chemotherapeutic agent that inhibits dihydrofolate reductase (choice C), an enzyme needed to produce purine nucleotides. Thus the "purine shortage" produced hinders cell replication. The Vinca alkaloids exert their effects by inhibiting microtubule formation (choice D).

Without microtubules, cells cannot mobilize their chromosomes, and thus mitosis is inhibited. A6 The correct answer is A. In the ego integrity vs. despair stage, individuals reconcile their achievements and failures and face the fact that any human life is limited. Persons completing this task find self-worth in reviewing their life events. Persons stuck at this task cannot admit that time has run out. Generativity vs. self-absorption (choice B) usually occurs from age 30-65. It involves noting ones contributions as a parent (in the broadest sense) and eventually "passing the torch" to the next generation. Persons stuck in this phase refuse to give up any power to their successors, and maintain ultimate authority without making a place for the next generation. Identity vs. role confusion (choice C) occurs during adolescence, and is usually defined by emotional and or geographic separation from the actual parents, with an internal identity based on both similarities and differences from parental traits and values. Industry vs. inferiority (choice D) is a stage in which school age (6-12) children explore peer groups and learn that industry (hard work) generally pays off. Intimacy vs. isolation (choice E) is the stage, in the early twenties, when young adults connect to their peers and significant others in a meaningful and mature way, putting aside fear and inhibitions.

Posted by admin at 8:14 AM 0 comments Labels: adenocarcinoma, Asbestos, Bronchioloalveolar carcinoma, Epinephrine, Hemoptysis, hoarseness, malignancy, Mesothelioma, Pulmonary fibrosis, Small cell carcinoma, Squamous cell carcinoma
SATURDAY, APRIL 10, 2010

Dysphagia case 4
A 35-year-old woman consults a physician because she has been having trouble swallowing. She also often experiences chronic heartburn. The physician performs a screening physical examination, and notices that the skin of her hands appears tight and

shiny. On specific questioning, she reports having often experienced color changes in her hands from white to blue to red. Q1 Which of the following is the most likely cause of the patient's difficulties with swallowing? / A. Achalasia / B. Adenocarcinoma / C. Chagas disease / D. Scleroderma / E. Squamous carcinoma Q2 The color changes described on the patient's hand are most likely due to which of the following? / A. Arteriolar spasm / B. Blood clots at sites of vascular injury / C. Large artery spasm / D. Platelet clots / E. Stasis blood clots Q3 Additional findings on physical examination include noting that the skin changes are limited to areas distal to the elbow and knee, the presence of calcified nodules on the extensor surfaces of the forearms, and the presence of telangiectasias on the forearms. This suggests that this patient has which of the following? / A. Bauer syndrome / B. Charcot syndrome / C. CREST syndrome / D. Crigler-Najjar syndrome / E. Dandy-Walker syndrome

Q4 If this woman's involved skin were biopsied, which of the following would most likely be seen? / A. Cleft separating the dermis and subcutaneous tissues / B. Epithelial cell hyperplasia / C. Marked dermal fibrosis / D. Narrowing of the basal lamina of small capillaries / E. Thickening of rete pegs Q5 More than 90% of the patients with the limited cutaneous form of this disorder make which of the following autoantibodies? / A. Anti-centromere / B. Anti-DNA topoisomerase l / C. Anti-double-stranded DNA / D. Anti-Golgi / E. Anti-ScI-70 Dysphagia case 4 answers A1 The correct answer is D. The tip-off is the reference to the patient's skin changes that are typical for scleroderma, also known as systemic sclerosis. Scleroderma is a disease that may be either predominately limited to the skin or involve many body systems, including the musculoskeletal system, gastrointestinal tract (with esophageal involvement most often symptomatic), cardiorespiratory system, and renal system. Esophageal dysfunction is a common complication of scleroderma. In most patients, the skin changes are obvious, even if the patient has not been previously diagnosed. Rarely, the skin changes may be noticed at an earlier stage, in which the skin of the hands appears puffy and edematous, but not scarred. Achalasia (choice A) and Chagas disease (choice C) are also important causes of dysphagia, but in achalasia, the physical examination is usually normal, and in Chagas disease, you should be able to elicit a history of possible exposure in Central or South America. Reflux esophagitis with risk of progression to Barrett esophagus and adenocarcinoma (choice B) can occur in scleroderma, but it is unlikely, at this early stage in the patient's

disease, that she already has adenocarcinoma. The risk of squamous cell carcinoma (choice E) is not increased in this patient. A2 The correct answer is A. The color changes described are typical for Raynaud's phenomenon, which occurs because of changes in perfusion due to arteriolar spasm. Raynaud's phenomenon is common in scleroderma, largely because the subintimal hyperplasia of small vessels characteristic of scleroderma can reduce the luminal diameter by more than 75%. Some authors argue that the vascular changes seen in scleroderma are actually the insult that triggers the subsequent development of fibrosis. Blood clots at sites of vascular injury (choice B) are typical in clots that form in arteries on surfaces such as a fractured atherosclerotic clot. Larger artery spasm (choice C) of vessels damaged by atherosclerosis is thought to contribute to some myocardial infarctions, but is not thought to be an important pathologic mechanism in scleroderma. Platelet clots (choice D) can be seen in some diseases in which platelet function is abnormal, such as essential thrombocytopenia. Stasis blood clots (choice E) commonly are found in venous thromboses. A3 The correct answer is C. These findings, together with esophageal dysfunction and Raynaud's phenomenon (both of which this patient has), are called the CREST syndrome, also known as limited cutaneous scleroderma. This form of scleroderma has a better longterm prognosis than when the skin changes also involve the trunk (diffuse scleroderma) and more internal organs are additionally involved. Bauer syndrome (choice A) is aortitis and aortic endocarditis as a complication of rheumatoid arthritis. Charcot syndrome (choice B) is intermittent claudication. Crigler-Najjar syndrome (choice D) is a severe familial liver disease. Dandy-Walker syndrome (choice E) is a malformation of the central nervous syndrome.

A4 The correct answer is C. In scleroderma, early changes (at the point at which the hands appear swollen, rather than with tight, thick skin) show edema with perivascular infiltrates of CD4+ T cells. At this stage, the collagen fibers are swollen and beginning to degenerate. The smaller vessels may show basal lamina thickening (not narrowing as in choice D) and endothelial (not epithelial cell hyperplasia) cell damage and proliferation. With time, the characteristic marked dermal fibrosis develops, which tends to both narrow (not thicken as in choice E) the rete pegs and attach the dermis tightly (compare with cleft formation as in choice A) to subcutaneous tissues. A5 The correct answer is A. All forms of scleroderma are thought to have a strong autoimmune component, and glucocorticoids and azathioprine are used to suppress the inflammatory complications of scleroderma. (Other drugs that can be used in therapy include penicillamine, which inhibits collagen cross-linking, NSAIDS for pain, and ACE inhibitors to protect the kidney if hypertension or renal damage occurs.) The anticentromere antibody is quite specific for CREST syndrome (96% of cases), and is only seen in a minority of patients with diffuse scleroderma (mainly those with Raynaud's phenomenon) and rarely in systemic lupus erythematosus and mixed connective tissue disease. Anti-DNA topoisomerase I (choice B), also called anti-Scl-70 (choice E) occurs commonly (64-75%) in diffuse scleroderma, but only rarely in CREST syndrome. Anti-double-stranded DNA (choice C) is fairly specific for systemic lupus erythematosus, although it only occurs in 50-60% of lupus cases. Anti-Golgi antibodies (choice D) are seen most often in systemic lupus erythematosus and Sjgren syndrome.

Dysphagia case 3
A 30-year-old man consults a physician because he has been having increasing difficulty swallowing both solids and liquids. Physical examination of the patient is noncontributory. Barium swallow studies show a mostly dilated esophagus with slow passage of barium into the stomach. The very distal part of the esophagus appears narrowed into a "bird's beak." Esophageal manometry shows incomplete relaxation of the lower esophageal sphincter in response to swallowing, high resting lower esophageal pressure, and absent esophageal peristalsis. Q1 The manometry and barium swallow studies most strongly support which of the following diagnoses? / A. Achalasia / B. Adenocarcinoma / C. Barrett esophagus / D. Squamous cell carcinoma / E. Systemic sclerosis Q2 This patient's condition is most likely due to which of the following? / A. Acid reflux / B. Cancerous destruction / C. Candida infection / D. Fibrosis of the esophageal wall / E. Lack of ganglion cells Q3 Which of the following regulators would most likely inhibit the lower esophageal sphincter in normal individuals? / A. Acetylcholine and substance P / B. Substance P and nitric oxide / C. Substance P only / D. Vasoactive intestinal polypeptide and acetylcholine / E. Vasoactive intestinal polypeptide and nitric oxide

Q4 Which of the following medications is used to directly relax the lower esophageal sphincter? / A. Diphenoxylate / B. Famotidine / C. Granisetron / D. Isosorbide dinitrate / E. Metoclopramide Q5 Worldwide, which of the following parasitic diseases is most likely to produce a disorder that clinically resembles this patient's condition? / A. Ascariasis / B. African sleeping sickness / C. Chagas disease / D. Cysticercosis / E. Malaria Dysphagia case 3 answers A1 The correct answer is A. The most likely diagnosis is achalasia. This condition is a neurogenic esophageal disorder that can occur at any age, but frequently is diagnosed when individuals are between the ages of 20 and 40. Characteristically, the swallowing difficulties involve both solid food and liquids. The manometry findings illustrated are typical; the barium swallow findings may be as illustrated or may instead show diffuse esophageal dilation without the "bird's beak" near the lower esophageal sphincter. Larger cancers of the esophagus (choices B and D) would be more likely to cause either a mass or an ulceration, which would be visible on barium swallow. Very small cancers and Barrett's esophagus (choice C) would require esophagogastroduodenoscopy with biopsy for diagnosis, and would be unlikely to cause dysphagia. Systemic sclerosis (choice E) can involve the esophagus as well, however, physical examination would usually show obvious skin involvement.

A2

The correct answer is E. Individuals who have achalasia have been found to have a deficiency of inhibitory ganglion cells within the esophageal wall. This lack causes an imbalance in excitatory and inhibitory neurotransmission, with the result that the lower esophageal sphincter tends to have a higher-than-normal muscle tone and relaxes only with difficulty. Acid reflux (choice A) can cause esophageal irritation, ulceration, and also predisposes for Barrett's metaplasia with subsequent risk of adenocarcinoma of the esophagus. Cancerous destruction (choice B) would produce a mass, or area of stricture or ulceration. Candida infection (choice C) of the esophagus resembles thrush of the mouth, and causes a usually superficial infection. Fibrosis of the esophageal wall (choice D) can be the consequence of ulceration (due to reflux or ingestion of harsh chemicals such as lye) or systemic sclerosis. A3

The correct answer is E. Physiologically important inhibitors of the lower esophageal sphincter include nitric oxide and vasoactive intestinal polypeptide. Physiologically important substances that stimulate the lower esophageal sphincter include acetylcholine and substance P. A4 The correct answer is D. Commonly used medications to relax the lower esophageal sphincter in patients with achalasia include nitrates such as isosorbide dinitrate (remember that nitric oxide physiologically inhibits the lower esophageal sphincter) and calcium channel blockers such as nifedipine (which inhibit calcium flow into the smooth muscle of the lower esophageal sphincter, thereby inhibiting contraction.) For patients in whom medical therapy fails, other options include paralysis of the lower esophageal sphincter with intrasphincteric injection of botulinum toxin, pneumatic dilatation, and a Heller myotomy (which interrupts the muscles of the lower esophageal sphincter). Diphenoxylate (choice A) is a opiate antidiarrheal, and would not be useful for achalasia. Famotidine (choice B) is an H2 antagonist that would be useful in reducing stomach acidity, but would not directly affect lower esophageal pressure. Other drugs in this class

include cimetidine, ranitidine, and nizatidine. Granisetron (choice C) is a 5HT3 antagonist and is used to prevent nausea and vomiting in patients who receive chemotherapy, and after general anesthesia. Other members of this drug class include ondansetron and dolasetron. Metoclopramide (choice E) stimulates gastric motility in patients with gastroparesis and is also a antiemetic agent. A5 The correct answer is C. Chagas disease, which is found in South and Central America and is due to infection with Trypanosoma cruzi, can involve the heart, colon, and esophagus. The esophageal involvement clinically closely resembles achalasia. The adult worms of ascariasis (choice A), or roundworm infection, live principally in the intestine, and can obstruct the intestine or a bile duct; the larvae can migrate to the liver, heart, and lungs. African sleeping sickness (choice B), caused by Trypanosoma brucei and Trypanosoma gambiense, causes lymphadenopathy, rash, and CNS involvement. Cysticercosis (choice D), due to the larval form of the pork tapeworm Taenia solium, can involve subcutaneous tissue, muscle, viscera (but not specifically the esophagus), and, most seriously, the CNS. Malaria (choice E), due to various Plasmodium species, involves the blood, liver, kidney, spleen, and brain, but does not have a specific predilection for the esophagus.

Dysphagia case 2
A 65-year-old man presents to his physician because he has been having increasing difficulty swallowing over the past 2 months. He is still able to swallow liquids, but swallowing solid food now causes severe pain and a sense of fullness behind his sternum. He has lost 18 pounds since his swallowing difficulties began. The patient is referred to a gastroenterologist, who demonstrates a mass lesion of the distal esophagus, which on biopsy is shown to contain cancer.

Q1 Which of the following is most important in separating the esophagus from the larynx, and must consequently be carefully passed behind during endoscopy? / A. Arytenoids / B. Cricoid cartilage / C. Epiglottis / D. Pharynx / E. Vocal cords Q2 Which of the following nerves provides the efferent impulses necessary for the esophageal actions that occur during swallowing? / A. Glossopharyngeal / B. Hypoglossal / C. Spinal accessory / D. Trigeminal / E. Vagus Q3 Which of the following approximately represents the proportion of different esophageal cancer types now being observed in the United States? / A. 1/10 adenocarcinoma and 9/10 squamous cell carcinoma / B. 1/3 adenocarcinoma and 2/3 squamous cell carcinoma / C. 1/2 adenocarcinoma and 1/2 squamous cell carcinoma / D. 2/3 adenocarcinoma and 1/3 squamous cell carcinoma / E. 9/10 adenocarcinoma and 1/10 squamous cell carcinoma

Q4 Precancerous metaplasia of the esophageal epithelium gives rise to a mucosa resembling which of the following? / A. Mesothelium / B. Respiratory epithelium / C. Small intestine / D. Squamous epithelium / E. Stomach Q5 Frequent use of which of the following has recently been found to probably have a protective effect against development of esophageal cancer? / A. Acetaminophen / B. Alcohol / C. Aspirin / D. Cigarettes / E. Codeine Q6 Currently, esophageal cancer has which of the following long-term survival rates? / A. Less than 5% / B. 30% / C. 50% / D. 70% / E. More than 95%

Dysphagia case 2 answers

A1 The correct answer is C. Endoscopists are very careful when guiding the endoscope past the epiglottis, which is a pear-shaped portion of elastic cartilage that can be moved during swallowing to close the larynx, preventing swallowed material from eventually entering the lungs. The arytenoids (choice A) are the site of the attachment of the vocal cords (choice E) within the larynx. The cricoid cartilage (choice B) is in the more distal portion of the larynx. The pharynx (choice D) is shared by the respiratory and gastrointestinal tracts. A2 The correct answer is E. The vagus nerve supplies the efferent input into the esophagus that is necessary for swallowing. The glossopharyngeal nerve (choice A) provides taste and sensation on the palate, but the only muscle it supplies is the stylopharyngeus. The hypoglossal nerve (choice B) moves the tongue during the initiation of swallowing, but does not innervate the esophagus. The spinal accessory nerve (choice C) plays no role in swallowing. This nerve mediates head and shoulder movement and innervates laryngeal muscles. The trigeminal nerve (choice D) provides general sensation to the mouth and motor innervation to the muscles of mastication. A3 The correct answer is C. More recent statistics indicate that the incidence of adenocarcinoma and squamous cell carcinoma of the esophagus are now roughly equal. Formerly, approximately 2/3 of the esophageal cancers were squamous in origin (choice B). Adenocarcinoma of the esophagus is often found in the distal esophagus.

A4 The correct answer is C. This is an indirect question about Barrett's esophagus, which is an important precursor of adenocarcinoma of the esophagus. While Barrett's esophagus was initially defined to be either gastric-type or intestinal-type metaplasia of the esophagus, more recent studies have shown that the actual problem lesion is more likely to be intestinal metaplasia (diagnosed when isolated goblet cells are seen in the epithelium) rather than gastric metaplasia (choice E). Metaplasia to mesothelium (choice A) or ciliated respiratory epithelium (choice B) does not usually occur in the esophagus. The normal epithelium of most of the esophagus is squamous (choice D). A5 The correct answer is C. An interesting new research observation that may be exploited in the future is that the incidence of esophageal cancer appears to be much lower in people who use aspirin frequently. Cigarettes (choice D) and alcohol use (choice B) have been implicated as risk factors for esophageal cancer. Acetaminophen (choice A) and codeine (choice E) have no known effects on the incidence of esophageal cancer. A6 The correct answer is A. Esophageal cancer is one of the very bad cancers, presently with poor long-term survival. The underlying problem is that the esophagus is only about 3 mm thick, and both metastatic disease and direct spread (often unresectable) to mediastinal structures is common. Active research is presently being undertaken to modify this prognosis by using chemotherapy and radiation therapy prior to surgery, but these modalities have not yet come into widespread use.

A 47-year-old, darkly pigmented man with a known history of alcohol abuse begins vomiting large quantities of blood and is brought by ambulance to the emergency department.

Q1

In the emergency department, the man is found to have a temperature of 36.7 C (98.1 F), blood pressure of 65/40 mm Hg and dropping rapidly, a weak pulse of 130/min, and respirations of 29/min. These vital signs suggest that which of the following is developing?
/ A. Congestive heart failure / B. Meningitis / C. Pneumonia / D. Septicemia / E. Shock Q2

A blood sample is drawn and an IV Iine is started. While the patient is being cross-matched, the physical examination is continued. The patient's sclerae are noted to be icteric and his nail beds and palms have a yellowish hue. A caput medusa is noted. Which of the following is the most accurate description of a caput medusa?
/ A. Ecchymoses over the mastoid process / B. Paradoxical increase in venous distension and pressure during inspiration / C. Reflex movement of the eyes in the opposite direction to that in which the head is moved / D. Small bony masses found on the terminal phalanges / E. Varicose veins radiating from the area of the umbilicus Q3

Caput medusa specifically suggests which of the following diagnoses?


/ A. BIadder infection

/ B. Duodenal ulcer / C. Gastric ulcer / D. Pancreatitis / E. Portal hypertension Q4

Which of the following is the most common cause of this patient's disorder in the United States?
/ A. Hepatic cirrhosis / B. Hepatic vein thrombosis / C. Hepatocellular carcinoma / D. Metastatic disease to the liver / E. Portal vein thrombosis Q5

Endoscopic studies demonstrate that this patient has bleeding esophageal varices, and the bleeding is successfully stopped with sclerotherapy. What percentage of patients with bleeding esophageal varices have another episode of variceal bleeding at a subsequent time?
/ A. 5% / B. 25% / C. 40% / D. 70%

/ E. 95% Q6

Following blood transfusions and sclerotherapy, the patient initially feels reasonably well and is able to converse with medical personneI. Over the next 12 hours, while he does not begin to rebleed, his mental status deteriorates. Arterial blood levels of which of the following would be most helpful in confirming the likely diagnosis?
/ A. Ammonia / B. Angiotensin l / C. Calcitonin / D. Carbon monoxide / E. Ceruloplasmin

Gastrointestinal Bleeding case 5 answers A1 The correct answer is E. The patient's low and dropping blood pressure, tachycardia, high respiratory rate, and slightly below normal body temperature are all consistent with impending shock. At this point, the other conditions listed in the choices have not yet been ruled out, but clinically, the patient should begin to be immediately treated for the shock, even if the therapeutic workup for underlying conditions must be temporarily deferred

A2

The correct answer is E. Medusa was a goddess with snakes instead of hair on her head. The caput medusa (Medusa's head) is an old term still in fairly common use for numerous varicose veins radiating over the abdomen from the area of the umbilicus. Choice A describes Battle's sign, which is suggestive of basal skull fracture. Choice B describes Kussmaul's sign, which is seen in constrictive pericarditis. Choice C describes the doll's eye sign, which is looked for in the evaluation of comatose patients and suggests functional integrity of the brainstem tegmental pathways and cranial nerves involved in eye movement.
Choice D describes Heberden's nodules, which are seen in osteoarthritis. A3 The correct answer is E. The caput medusa develops when severe portal hypertension induces dilation of the anastomotic channels between the portal venous system and the systemic venous system, some of which involve the superficial veins near the umbilicus. The other answers are distracters. A4 The correct answer is A. The overwhelmingly most common cause of portal hypertension in the United States is hepatic cirrhosis, which is usually due to either alcoholism or hepatitis viral infection. In this patient's case, the diagnosis of cirrhosis is further clinically substantiated by his jaundice, as evidenced by his sclera, nail beds, and palms. (Look in these areas on individuals in whom dark skin pigmentation may mask the jaundice generally.) The other entities listed are occasional causes of portal hypertension. A5 The correct answer is D. Patients who have had one episode of bleeding from esophageal varices have an approximately 70% chance of developing a second incident of bleeding, and one third of these episodes of rebleeding is fatal.

A6

The correct answer is A. Hepatic encephalopathy is seen in end-stage cirrhosis patients, and can either present or worsen in the presence of gastrointestinal bleeding. The blood in the upper gastrointestinal tract behaves essentially as a high protein load, and increases the absorption of ammonia and nitrogen, which cannot be appropriately metabolized by the liver. GI bleeding may also predispose for inadequate renal function secondary to hypotension. Angiotensin I (choice B) is part of the renin-angiotensin-aldosterone system for blood pressure and sodium ion control. Calcitonin (choice C) is a hormone secreted by the thyroid, which may be increased in medullary carcinoma of the thyroid. Carbon monoxide (choice D) increases in the blood in smokers.
Ceruloplasmin (choice E) is a copper-carrying protein monitored in patients with Wilson disease.

Hemoptysis Case 3
A 53-year-old man consults a physician because he has begun coughing up sputum tinged with fresh blood. He does not initially report any other symptoms to his physician. When his physician comments on his hoarseness and cough, the patient discounts these symptoms and attributes them to his long smoking history. He cannot say when they began or became worse. Q1 The chronic hoarseness suggests dysfunction of which of the following? / A. Palate / B. Pharynx / C. Tongue / D. Trachea / E. Vocal cords

Q2 Laryngoscopy reveal a fungating tumor of the larynx that is located between the false and true vocal cords. This tumor is in which of the following sites? / A. Aryepiglottic fold / B. Infraglottic compartment / C. Piriform recess / D. Supraglottic compartment / E. Ventricle Q3 Biopsy of the mass demonstrates a malignancy. Which of the following is the most likely diagnosis? / A. Adenocarcinoma / B. Lymphoma / C. Oat cell carcinoma / D. Sarcoma / E. Squamous cell carcinoma Q4 In addition to smoking, which of the following is an accepted risk factor for this patient's tumor? / A. Alcohol use / B. Cocaine use / C. Coffee use / D. Marijuana use / E. Tea use

Q5 The patient returns to clinic three weeks after receiving the news that his cancer is inoperable. His wife reports that he has been more withdrawn, eating and sleeping poorly and "just seems to have lost all hope." On examination, the patient moves very little, never makes eye contact, and admits to "some" depressed mood, Ioss of appetite, and sleep disruption. Which of the following symptoms is of most concern regarding his risk of suicide? / A. Decreased appetite / B. Decreased energy / C. Diminished concentration / D. Guilty and worthless feelings / E. Hopelessness Q6 Some patients present with hoarseness due to a tumor at the lung apex that involves a nerve that is a branch of which of the following? / A. Accessory nerve / B. Glossopharyngeal nerve / C. Hypoglossal nerve / D. Phrenic nerve / E. Vagus nerve Hemoptysis Case 3 Answers A1 The correct answer is E. Hoarseness specifically suggests dysfunction of the vocal cords, which produce the sounds that are then articulated to speech with the pharynx (choice B), palate (choice A), tongue (choice C), teeth, and lips. While vocal cord dysfunction is specifically suggested by hoarseness, the anatomic lesion may or may not be present at the level of the vocal cords. Vocal paralysis may be the result of local tumor or trauma, intracranial lesions affecting the nucleus ambiguus or its supranuclear tracts, and lesions at the base of the skull, neck, or upper portion of the thorax that involve either the vagus nerve or the recurrent laryngeal nerves. Air from the trachea (choice D) passes into the larynx to reach the vocal cords, but tracheal dysfunction does not produce hoarseness.

A2 The correct answer is E. The interior of the larynx is divided into 3 compartments: the supraglottic compartment (choice D) above the false vocal cords, the ventricle between the false and true vocal cords, and the infraglottic compartment (choice B) below the true vocal cords. The aryepiglottic fold (choice A) is the upper free border of the quadrangular membrane found in the supraglottic compartment. The piriform recess (choice C) lies behind the thyroid laminae and the lateral wall of the supraglottic compartment. A3 The correct answer is E. The epithelial lining of the larynx is squamous epithelium, and the vast majority of cancers of the larynx are squamous cell carcinomas. Squamous cell carcinoma can present with hoarseness, cough, hemoptysis, or difficulty swallowing. If the symptoms develop insidiously (as in the case), the patient may not seek medical attention until late in the course. While very small laryngeal cancers can be treated successfully with surgery and/or radiation, larger ones are much more problematic, in large part because complete resection of the cancer and any lymph node metastases can be difficult to impossible to perform without compromising the many vital neck structures. Patients treated with partial laryngectomy may retain some speech ability. Patients treated with total laryngectomy can often learn to speak again using esophageal speech (gradual belching of air through the pharyngoesophageal junction), a tracheoesophageal fistula (one way valve between the trachea and the esophagus which makes a sound when air is forced across it), or an electrolarynx (sound source held against the neck). With all three techniques, the sound produced is then turned into articulation by the patient's pharynx, palate, tongue, teeth, and lips. The other tumors listed in the choices are very uncommon in the larynx. A4 The correct answer is A. The only two risk factors that you will need to associate with laryngeal carcinoma are smoking and alcohol use. Cocaine (choice B), coffee (choice C), and tea (choice E) have not been linked to laryngeal cancer. Marijuana (choice D) has been suggested as a risk factor for oral cancer in a few cases, but a causal association has not been established.

A5 The correct answer is E. Hopelessness is an ominous sign and is associated with a higher risk for suicide. Decreased appetite (choice A), decreased energy (choice B), and diminished concentration (choice C) are symptoms indicating the presence of a major depressive episode, or are the result of the medical illness. Guilty and worthless feelings (choice D) are symptoms of depression. They do not predict risk for suicide.

A6 The correct answer is E. The larynx has complex innervation. The vocal cords and most of the area of the larynx below them are supplied by the recurrent laryngeal nerve, which is a branch of the vagus nerve that passes beneath the subclavian artery before returning to the neck to innervate the larynx. From above the larynx, the superior laryngeal nerve arises from the vagus and divides into the internal laryngeal nerve and the external laryngeal nerve. The innervation above the vocal cords is by the internal laryngeal branch of the vagus nerve. The cricothyroid muscle (the only muscle of the larynx not supplied by the recurrent laryngeal nerve) is supplied by the external laryngeal branch of the superior laryngeal, which also branches off the vagus, but which contains motor fibers originally derived from the accessory nerve (choice A). The glossopharyngeal nerve (choice B) supplies the pharynx. The hypoglossal nerve (choice C) supplies the tongue. The phrenic nerve (choice D) supplies the diaphragm.

Dysphagia case 2
A 65-year-old man presents to his physician because he has been having increasing difficulty swallowing over the past 2 months. He is still able to swallow liquids, but swallowing solid food now causes severe pain and a sense of fullness behind his sternum. He has lost 18 pounds since his swallowing difficulties began. The patient is referred to a gastroenterologist, who demonstrates a mass lesion of the distal esophagus, which on biopsy is shown to contain cancer.

Q1 Which of the following is most important in separating the esophagus from the larynx, and must consequently be carefully passed behind during endoscopy? / A. Arytenoids / B. Cricoid cartilage / C. Epiglottis / D. Pharynx / E. Vocal cords Q2 Which of the following nerves provides the efferent impulses necessary for the esophageal actions that occur during swallowing? / A. Glossopharyngeal / B. Hypoglossal / C. Spinal accessory / D. Trigeminal / E. Vagus

Q3 Which of the following approximately represents the proportion of different esophageal cancer types now being observed in the United States? / A. 1/10 adenocarcinoma and 9/10 squamous cell carcinoma / B. 1/3 adenocarcinoma and 2/3 squamous cell carcinoma / C. 1/2 adenocarcinoma and 1/2 squamous cell carcinoma / D. 2/3 adenocarcinoma and 1/3 squamous cell carcinoma / E. 9/10 adenocarcinoma and 1/10 squamous cell carcinoma

Q4 Precancerous metaplasia of the esophageal epithelium gives rise to a mucosa resembling which of the following? / A. Mesothelium / B. Respiratory epithelium / C. Small intestine / D. Squamous epithelium / E. Stomach Q5 Frequent use of which of the following has recently been found to probably have a protective effect against development of esophageal cancer? / A. Acetaminophen / B. Alcohol / C. Aspirin / D. Cigarettes / E. Codeine Q6 Currently, esophageal cancer has which of the following long-term survival rates? / A. Less than 5% / B. 30% / C. 50% / D. 70% / E. More than 95% Dysphagia case 2 answers A1 The correct answer is C. Endoscopists are very careful when guiding the endoscope past the epiglottis, which is a pear-shaped portion of elastic cartilage that can be moved during swallowing to close the larynx, preventing swallowed material from eventually entering the lungs. The arytenoids (choice A) are the site of the attachment of the vocal cords (choice E) within the larynx. The cricoid cartilage (choice B) is in the more distal portion of the larynx.The pharynx (choice D) is shared by the respiratory and gastrointestinal tracts.

A2 The correct answer is E. The vagus nerve supplies the efferent input into the esophagus that is necessary for swallowing. The glossopharyngeal nerve (choice A) provides taste and sensation on the palate, but the only muscle it supplies is the stylopharyngeus. The hypoglossal nerve (choice B) moves the tongue during the initiation of swallowing, but does not innervate the esophagus. The spinal accessory nerve (choice C) plays no role in swallowing. This nerve mediates head and shoulder movement and innervates laryngeal muscles. The trigeminal nerve (choice D) provides general sensation to the mouth and motor innervation to the muscles of mastication. A3 The correct answer is C. More recent statistics indicate that the incidence of adenocarcinoma and squamous cell carcinoma of the esophagus are now roughly equal. Formerly, approximately 2/3 of the esophageal cancers were squamous in origin (choice B). Adenocarcinoma of the esophagus is often found in the distal esophagus.

A4 The correct answer is C. This is an indirect question about Barrett's esophagus, which is an important precursor of adenocarcinoma of the esophagus. While Barrett's esophagus was initially defined to be either gastric-type or intestinal-type metaplasia of the esophagus, more recent studies have shown that the actual problem lesion is more likely to be intestinal metaplasia (diagnosed when isolated goblet cells are seen in the epithelium) rather than gastric metaplasia (choice E). Metaplasia to mesothelium (choice A) or ciliated respiratory epithelium (choice B) does not usually occur in the esophagus. The normal epithelium of most of the esophagus is squamous (choice D).

A5 The correct answer is C. An interesting new research observation that may be exploited in the future is that the incidence of esophageal cancer appears to be much lower in people who use aspirin frequently. Cigarettes (choice D) and alcohol use (choice B) have been implicated as risk factors for esophageal cancer. Acetaminophen (choice A) and codeine (choice E) have no known effects on the incidence of esophageal cancer. A6 The correct answer is A. Esophageal cancer is one of the very bad cancers, presently with poor long-term survival. The underlying problem is that the esophagus is only about 3 mm thick, and both metastatic disease and direct spread (often unresectable) to mediastinal structures is common. Active research is presently being undertaken to modify this prognosis by using chemotherapy and radiation therapy prior to surgery, but these modalities have not yet come into widespread use.

A 55-year-old man with a history of coronary artery disease and alcoholism presents to the emergency department complaining that he vomited bright red blood twice this morning. He denies previous episodes of bleeding or abdominal pain. On examination, he is a malnourished man in acute distress. His blood pressure is 90/50 mm Hg and his pulse is 110/min. His mucous membranes are dry and his sclera are icteric. Abdominal examination reveals a distended abdomen with an enlarged, palpable spleen. Purplish striae are seen around the umbilicus. On rectal examination, Iarge hemorrhoids are seen, but the stool is negative for blood.

Q1

Which of the following is the most likely diagnosis?


/ / / / /

A. Erosive gastritis B. Esophageal varices C. Infectious enteritis D. Mallory Weiss tear E. Peptic ulcer disease

Q2

Which of the following coagulation factors would most likely be unaffected in this patient?
/ / / / /

A. Factor ll B. Factor VII C. Factor IX D. Factor XIII E. Von Willebrand's factor

Q3

Which of the following anatomic relationships provides the basis for the patient's hemorrhoids?
/ / / / /

A. Coronary vein anastomosis with the esophageal plexus B. Inferior rectal vein anastomosis with the iliac vein C. Paraumbilical vein anastomosis with the inferior epigastric vein D. Superior mesenteric vein anastomosis with the splenic vein E. Superior rectal vein anastomosis with the inferior and middle rectal vein

Q4

Which of the following structures are found in the portal triad?


/ / / / /

A. Hepatic vein, common hepatic artery, common bile duct B. Portal vein, celiac artery, common bile duct C. Portal vein, common hepatic artery, common bile duct D. Portal vein, falciform ligament, common bile duct E. Portal vein, sinusoids, bile canaliculi

Gastrointestinal Bleeding case 3 answers A1


The correct answer is B. While all of the answer choices listed must be considered in the differential, upper gastrointestinal bleeding from esophageal varices is most likely. This patient displays many of the stigmata of hepatic disease and portal hypertension: icteric sclera, hemorrhoids, distended umbilical veins (caput medusae), and a history of

alcoholism. In this setting, esophageal varices would be the most likely. To make this diagnosis definitively, however, one needs to examine the gastrointestinal tract endoscopically. Erosive gastritis (choice A) is a source of upper gastrointestinal hemorrhage, but it seldom bleeds so profusely that the patient becomes hemodynamically unstable. Infectious disease in the gastrointestinal tract (choice C) may produce hemorrhage, but it tends to produce lower GI bleeding. Mallory Weiss tears (choice D) produce upper GI bleeding. This tearing of the gastroesophageal junction occurs in alcoholics, but usually a history of retching precedes bleeding. No such history is elicited here.

Peptic ulcer disease (choice E) can produce brisk upper GI bleeding. It is less likely in this case because this patient has no history of GI pain. A2
The correct answer is E. Von Willebrand's factor is a coagulation factor produced by the vascular endothelium and megakaryocytes. It is the only protein in the cascade that is not synthesized in the liver. vWF mediates the adhesion of platelets to the vessel wall basement membrane after vascular injury. Patients with a deficiency of von Willebrand's factor have a tendency to bleed. It is an autosomal dominant disease, and the ristocetin cofactor activity test is the best way to clinically assess vWF function. Factor II (choice A) is produced in the liver. Deficiency is very rare, but can produce spontaneous or posttraumatic bleeding. Factor VII (choice B) is produced in the liver.Severe factor VII deficiency is a very rare cause of bleeding. Factor IX (choice C) is produced in the liver. A factor IX deficiency is known as hemophilia B, which is an X-linked disease.

Factor XIII (choice D) is produced in the liver. A deficiency of factor XIII produces delayed bleeding and poor wound healing.

A3
The correct answer is E. The patient's hemorrhoids are a consequence of his portal hypertension. The patient has a cirrhotic liver, which impedes circulation in the portal system. As the pressure rises in the portal system, blood in the portal circulation begins to backflow into the caval circulation. At the sites at which the portal system anastomoses with the caval circulation, venous engorgement occurs. At one such site, the confluence of the superior rectal vein (portal) with the middle and inferior rectal vein (caval), this venous engorgement leads to hemorrhoids. The coronary vein anastomosis with the esophageal venous plexus (choice A) provides the anatomic basis for the esophageal varices seen in portal hypertension. As pressure builds in the portal system, venous engorgement occurs, and varices are produced in the distal esophagus. These varices can be the site of life-threatening upper GI bleeding. The anastomosis of the inferior rectal vein with the iliac vein (choice B) is a caval-caval anastomosis and would not be affected by portal hypertension. The anastomosis of the paraumbilical vein and the inferior epigastric vein (choice C) is the portal-caval anastomosis responsible for the purplish striae or caput medusae seen on this patient's abdomen. This circulatory route is an embryologic remnant, and is only patent when portal pressure rises high enough to re-open this pathway.

The anastomosis of the superior mesenteric vein and the splenic vein (choice D) marks the origin of the portal vein. It may have an elevated pressure, but it is not the basis for hemorrhoids. A4
The correct answer is C. The portal triad contains the portal vein, common hepatic artery, and common bile duct. It is found in the fold of peritoneum, called the hepatoduodenal ligament, that separates the greater and lesser abdominal sacs. None of the other choices offer a complete answer: The hepatic vein (choice A) drains the liver into the inferior vena cava (IVC). The celiac artery (choice B) supplies blood to the anatomic foregut. One of its branches, the common hepatic artery, travels in the porta hepatis.

The falciform ligament (choice D) is the remnant of the umbilical vein that passes from the anterior abdominal wall to the superior surface of the liver.

Bile canaliculi (choice E) are microscopic channels that drain bile from the hepatocytes. A 64-year-old man with a history of coronary artery disease (CAD) comes to the emergency department with the acute onset of severe, constant, Lower abdominal pain and rectal bleeding. He reports that he previously has had several episodes of similar, but less severe pain. About 12 hours after the onset of pain, the patient began passing copious bright red blood per rectum. He denies nausea, vomiting, sick contacts, or foreign traveI. Initial physical examination reveals a distressed man, who is afebrile, but tachypneic, with scant diffuse abdominal tenderness to palpation. Rectal examination is positive for blood. Laboratory studies reveal a metabolic acidosis with an elevated serum Iactate.
Q1

Which of the following is the most likely diagnosis?


/ / / / /

A. Colon carcinoma B. Infectious colitis C. Inflammatory bowel disease D. Ischemic colitis E. Necrotizing enterocolitis

Q2

The lactate produced from the anaerobic metabolism in the infarcted gut will likely be which of the following?
/ /

A. Exhaled as a fruity odor B. Incorporated into glycogen in the liver

/ / /

C. Incorporated into myoglobin in muscle D. Incorporated into urea in the urine E. Secreted by the kidneys unchanged

Q3

If this patient's disease were drug-induced, which of the following agents would most likely be responsible?
/ / / / /

A. Acetaminophen B. Amiodarone C. Cocaine D. Dexamethasone E. Nitroglycerin

Q4

While the patient is in the emergency department, the pain becomes increasingly severe. Several hours after his initial examination, the patient becomes febrile and is now exquisitely tender to palpation. He writhes in pain when the physician jostles the bed. Air is seen under the diaphragm in an upright chest x-ray film. These new findings suggest which of the following?
/ / / / /

A. Abdominal aortic aneurysm B. Bowel obstruction C. Cholecystitis D. Hypovolemia E. Perforation with peritonitis

Q5

Upon surgical exploration of the abdomen, the colon is dull and dusky from the mid transverse colon to the rectum. The patient has occluded which of the following vessels?
/ / / / /

A. Celiac trunk B. Cystic artery C. External iliac artery D. Inferior mesenteric artery E. Superior mesenteric artery

______________________________________________________________________ ______ Abdominal Pain Case 3 Answers


A1 The correct answer is D. A patient with severe abdominal pain and rectal bleeding with an unremarkable physical examination is likely suffering from ischemic colitis. "Pain outof-proportion to examination" is a classic finding for ischemic colitis. The previous episodes of less severe pain represent ischemic angina. An infarction has occurred, as indicated by the rise in serum lactate secondary to the colon's anaerobic metabolism. The history of coronary artery disease also suggests this diagnosis, as the atherosclerotic processes that contribute to his CAD are also likely present in his abdominal vasculature. Colon cancer (choice A) would produce less acute symptoms, but occasionally, colon cancer may present acutely with obstructive symptoms. Patients may have bleeding and abdominal pain, but the pain is typically intermittent and accompanied by nausea, vomiting, abdominal distention, and absence of flatus.

Infectious colitis (choice B) is incorrect. While patients may have bleeding and abdominal pain, nothing in the history suggests a disease of infectious origin (no sick contacts or foreign travel). The acute onset also suggests a vascular event, rather than an infectious one. Inflammatory bowel disease (IBD) (choice C) is incorrect because while the patient reports previous episodes, an elderly man with IBD would likely have a chronic history of abdominal pain and bleeding.

Necrotizing enterocolitis (choice E) affects premature infants and would not be relevant in this setting. A2
The correct answer is B. Lactate is converted into glucose, and then glycogen in the liver by a process know as the Cori cycle. Choice A is incorrect, as lactate would not be exhaled. A fruity odor on the breath would be a sign of ketoacidosis. While some of the carbon from the lactate may be incorporated into peptides via Krebs intermediates (e.g., choice C), the vast majority would be left as carbohydrate. Urea (choice D) represents a means of eliminating nitrogenous waste.

Choice E is wrong, as the kidneys would retain the lactate, rather than excreting it. A3
The correct answer is C. Cocaine is a sympathomimetic drug that indirectly acts on both the alpha and beta adrenergic receptors on the vasculature. As such, cocaine may cause vasospasm in the abdominal vasculature leading to infarction and ischemic colitis. Similar vasospastic events may occur in the coronary vasculature, leading to myocardial infarction. Acetaminophen (choice A) is an analgesic, and would not play a role in producing ischemic colitis.

Amiodarone (choice B) is an antiarrhythmic, and would not contribute to ischemic colitis. Dexamethasone (choice D) is a steroidal anti-inflammatory drug. Not only would this medication not cause ischemic colitis, it might mask the symptoms due to its potent antiinflammatory properties.

Nitroglycerin (choice E) is a venodilator, and would not contribute to ischemic colitis. As a venodilator, nitroglycerin is used to treat coronary ischemia by reducing cardiac preload. A4
The correct answer is E. This patient has experienced a bowel perforation. Air under the diaphragm in an upright chest film provides definitive evidence that a hollow viscus has ruptured. Air near the liver on a left lateral decubitus (patient lays with the left side down) is an alternative study to demonstrate perforation. Spillage from the perforated bowel has irritated and inflamed the peritoneum, resulting in peritonitis. Symptoms of peritonitis include extreme, sharp pain exacerbated by jostling (patients often report that the bumpy ride to the emergency department caused extreme pain). Patients will be exquisitely tender to palpation and percussion and may have abdominal rigidity. Fever typically accompanies peritonitis. While an abdominal aortic aneurysm or AAA (choice A) presents as acute abdominal pain, this pain is described as tearing and may radiate to the back. A pulsatile abdominal mass may be palpated. The air on the chest film is also inconsistent with AAA. This patient does not have bowel obstruction (choice B). Signs and symptoms of bowel obstruction include: nausea, vomiting, intermittent abdominal pain, hypovolemia, abdominal distention, absence of flatus, and a "step ladder" bowel pattern on abdominal films. Cholecystitis (choice C) typically presents as right upper quadrant (RUQ) pain, fever, and jaundice. Patients usually have a history of colicky RUQ pain.

While the patient is at risk for hypovolemia (choice D), none of the symptoms listed typify hypovolemia. Signs and symptoms of mild to moderate hypovolemia include malaise, dry mouth, thirst, decreased skin turgor, tachycardia, hypotension, and decreased urine output. A5

The correct answer is D. The inferior mesenteric artery distributes blood to the embryologic hindgut. This includes the distal 1/3 of the transverse colon to the rectum. The rectum is spared because it receives circulation from the inferior rectal artery (not mesenteric). The celiac trunk (choice A) supplies the embryologic foregut. The first three branches include the splenic artery, the left gastric artery, and the common hepatic artery. This patient has no findings in this distribution. The cystic artery (choice B) supplies the gall bladder. There are no gall bladder findings in this case. The external iliac artery (choice C) gives rise to the vessels of the lower extremity. Symptoms of occlusion or stenosis might include buttock and thigh pain exacerbated by walking. Severe stenosis might give patients buttock and thigh pain, even at rest.

The superior mesenteric artery (choice E) supplies the embryologic hindgut. This extends from the duodenum to the proximal 2/3 of the transverse colon
50-year-old man consults a physician because he has developed a chronic, non-productive cough and is experiencing a reduced ability to do strenuous work. His symptoms have developed insidiously. On questioning, he states that he is a smoker and has also worked as a contractor for all of his adult life. Physical examination is notable for the presence of repetitive end-inspiratory basal crackles and finger clubbing. A chest x-ray film shows diffusely distributed, small irregular opacities that are most prominent in the lower lung zones. Localized areas of pleural thickening are also noted. No large masses are seen. Q1 The chest x-ray film is most consistent with which of the following? / A. Emphysema / B. Interstitial disease / C. Lobar pneumonia / D. Lung cancer / E. Pleural effusion Q2

The patient is sent for spirometry for further evaluation. FEV1 and FVC are both shown to be about 60% of the expected values, and the ratio of FEV1/FVC is 90%. These findings are most consistent with which of the following? / A. Asthma / B. Bronchiectasis / C. Chronic bronchitis / D. Emphysema / E. Restrictive lung disease

Q3 The patient's work history is most suggestive of exposure to which of the following? / A. Asbestos / B. Beryllium / C. Coal / D. Kaolin / E. Silica Q4 In addition to predisposing for pulmonary fibrosis and bronchogenic carcinoma, this patient's disease is associated which of the following? / A. Basal cell carcinoma / B. Germ cell tumor / C. Hemangioma / D. Mesothelioma / E. Pleomorphic adenoma ____________________________________________________________________ Cough Case 1 Answers A1 The correct answer is B. Diffusely distributed small irregular opacities suggest the presence of interstitial lung disease. Emphysema (choice A) would produce unusually dark lung fields. Lobar pneumonia (choice C) would produce a "white out" of one or more lung lobes. Lung cancer (choice D), if large, would produce a mass lesion (often involving a bronchus), or, if very small, might not be recognized on chest x-ray. Pleural effusion (choice E) would cause a whitened area due to fluid below the lung A2

The correct answer is E. Spirometry is commonly used to subdivide non-tumorous lung diseases into diseases that are predominately obstructive in nature and those that are predominately restrictive. The process of spirometry involves controlled breathing in and out while airflow is plotted against volume to obtain a continuous loop. FVC (forced vital capacity) is the maximum volume of air that can be forcibly and rapidly exhaled following a maximum inspiration. FEV1 (forced expiratory volume in the first second) is the volume of air expelled in the first second of a forced expiration starting from full inspiration. Restrictive lung disease usually shows a reduction in both FVC and FEV1, and the ratio of FEV1/FVC is greater than 80%. If these readings are obtained in a patient, further studies to measure lung volumes are usually performed to confirm the finding. Asthma (choice A), bronchiectasis (choice B), chronic bronchitis (choice C), and emphysema (choice D) all tend to produce obstructive patterns, with FEV1 disproportionately decreased when compared to FVC.

A3 The correct answer is A. Working history often offers helpful clues about possible toxic exposures that may have contributed to lung disease. Asbestos was formerly a common constituent of insulating material in buildings because of both its insulating properties and its fire-resistant properties. People working on old buildings, particularly when removing the old insulation, are consequently vulnerable to high exposures unless they take care to minimize exposure with respirators. The risk to individuals living and working in old buildings is usually markedly less, since the asbestos is typically found behind walls. Individuals who work in asbestos mines (principally in Canada, South Africa, and the former USSR) may also have high exposures. Beryllium (choice B) is used in the nuclear industry and in x-ray tubes, and was formerly used in ceramics, metallic alloys, and fluorescent lights. Most significant coal (choice C) exposures occur in coal miners. Kaolin (choice D) is a component of clay dust. Silica (choice E) is found in sand and glass, and significant exposures can be seen in individuals working in environments where small particles of these materials may become aerosolized. A4 The correct answer is D. Asbestos exposure is also linked to late (often after 20 years) development of the rare tumor, malignant mesothelioma. The amphibole forms of asbestos appear to be much more likely to induce mesothelioma than does chrysotile, and some authors have even speculated that the rare cases of mesothelioma in persons with predominant exposure to the serpentine form of asbestos may have been actually related to trace exposures to the amphibole forms. In contrast, all types of asbestos can cause

pleural plaques, pulmonary fibrosis, and lung cancer. None of the other lesions listed in the choices have ties to asbestos. Basal cell carcinoma (choice A) is a form of skin cancer. Germ cell tumors (choice B) usually occur in testes and ovary. Hemangioma (choice C) is a benign tumor of blood vessels. Pleomorphic adenoma (choice E) is a salivary gland tumor.

A 14-year-old girl receives a bone marrow transplant as part of her treatment for acute lymphoblastic lymphoma. During the period of profound immunosuppression before the marrow engrafts, she develops nonproductive cough, fever, mild hemoptysis, and pleuritic chest pain. A plain chest x-ray film shows a pleuraI-based wedge-shaped lesion with focal cavitation. Open chest lung biopsy reveals necrosis and hemorrhage. Septate fungal forms with dichotomous 45-degree branching are seen in the necrotic areas and involving the walls of several blood vessels. When the fungus is cultured, it is found to be a monomorphic fungus. Q1 Which of the following is a monomorphic filamentous fungus? / A. Aspergillus / B. Blastomyces / C. Coccidides / D. Histoplasma / E. Sporothrix Q2 Two commonly encountered invasive fungi are Aspergillus and Candidia. In tissue specimens, which of the following features is often helpful in distinguishing these organisms? / A. Aspergillus has both hyphae and pseudohyphae / B. Aspergillus has budding yeasts / C. Aspergillus has germ tubes / D. Aspergillus shows dichotomous branching

/ E. Aspergillus shows generally obtuse angles of branching Q3 Involvement by this patient's infection of which of the following is considered to have the worst prognosis? / A. Brain / B. Lung / C. Middle ear / D. Sinus / E. Skin Q4 Which of the following is the most appropriate pharmacotherapy for this patient? / A. Amphotericin B / B. Clotrimazole / C. Fluconazole / D. Flucytosine / E. Griseofulvin Q5 Which of the following best describes the mechanism of action of the most appropriate medication for this patient's disease? / A. Forms pores in fungal membranes / B. Inhibits the demethylation of lanosterol / C. Inhibits squalene epoxidase / D. Inhibits thymidylate synthase / E. Interferes with the synthesis of ergosterol Q6 Toxicity to which of the following organs is most likely to limit the administration of the most appropriate medication for this patient's disease? / A. Brain / B. Heart

/ C. Kidney / D. Liver / E. Lung ____________________________________________________________________ Cough Case 5 Answers A1 The correct answer is A. The fungi that cause "deep infections" in humans are subdivided into the dimorphic forms (which, depending upon temperature, can be either yeast forms or hyphal forms) and the monomorphic forms (which grow in the same general form at different temperatures). Of the fungi listed, only Aspergillus is monomorphic. Aspergillus is a common saprophytic mold found on decaying material in the environment throughout the world. It can cause human diseases, including allergic bronchopulmonary aspergillosis (which is essentially an allergic reaction to inhaled Aspergillus conidia or spores), fungus ball (in which the Aspergillus grows without invading in a preexisting cavitary lesion of the lung), invasive aspergillosis (including pneumonia, meningitis, and other systemic infections), and cellulitis. Invasive aspergillosis is most often seen in severely immunocompromised patients with severe neutropenia, notably including those with a history of transplantation, chronic granulomatous disease, and leukemia. This patient's presentation is typical for invasive pulmonary aspergillosis. The other fungi listed (choices B, C, D, and E) are all dimorphic. A2 The correct answer is D. Aspergillus has hyphae, but not pseudohyphae, budding yeasts, or germ tubes. The branching is dichotomous and at an acute (often about 45 degrees) angle (opposite of choice E). Characteristics to look for with Candida include both pseudohyphae and true hyphae (choice A), budding yeasts (choice B), and occasionally germ tubes (choice C, better seen with some culture methods). A3 The correct answer is A. Cerebral aspergillosis is a feared complication of other forms of aspergillosis because most patients die despite appropriate antifungal therapy. Of the other sites listed in the question choices, true invasive infection of the lung (choice B), as opposed to a fungus ball, is considered the most serious site of infection, though less so

than infection of the brain. Middle ear and sinus involvement (choices C and D) become clinically worrisome when dissemination or extension to the brain occurs. Skin involvement (choice E) is worrisome if dissemination occurs. A4 The correct answer is A. Amphotericin B is the mainstay of therapy of invasive aspergillosis. This medication has a wide fungicidal spectrum and remains either the drug of choice or the co-drug of choice for severe infections caused by Aspergillus, Candida, Cryptococcus, Histoplasma, Mucor, and Sporothrix. Alternative antifungal agents that are not the preferred therapy but do have some activity against aspergillus include itraconazole, flucytosine (choice D), and voriconazole. Clotrimazole (choice B) is used topically for candidal and dermatophytic infections. Fluconazole (choice C) is the drug of choice for esophageal and invasive candidiasis and coccidioidomycosis, and is used for prophylaxis and suppression in cryptococcal meningitis. Flucytosine (choice D) is also synergistic with amphotericin B in candidiasis and cryptococcosis. Griseofulvin (choice E) is given orally, but is active only against dermatophytes. A5 The correct answer is A. The polyene anti-fungal amphotericin B is an amphoteric compound, with both polar and non-polar structural elements, that interacts with ergosterol in fungal membranes to form artificial "pores," which disrupt membrane permeability. Resistant fungal strains are those that appear to have low ergosterol content in their cell membranes. Amphotericin B is given by slow IV infusion and penetrates poorly into the central nervous system (intrathecal injections can be used). It has a half-life of more than 2 weeks and is removed by both metabolism and renal elimination. Antifungal agents in the azole class (including ketoconazole, fluconazole, and itraconazole) interfere with the synthesis of ergosterol (choice E) by inhibiting the P450dependent 14-alpha-demethylation of its precursor molecule, lanosterol (choice B). Flucytosine is activated by fungal cytosine deaminase to 5-fluorouracil (5-FU), which can be incorporated into fungal RNA and can be also used to form 5-fluorodeoxyuridine monophosphate, which in turn inhibits thymidylate synthase (choice D) and the synthesis of thymine.

The anti-dermatophytic drug terbinafine inhibits squalene epoxidase (choice C) and thus decreases ergosterol synthesis (choice E). A6 The correct answer is C. Nephrotoxicity is most likely to be dose-limiting, and may force amphotericin treatment to end prematurely. This is an important problem, since we have so few drugs with wide spectrum activity against invasive fungal organisms. An amphotericin B lipid complex (ABLC, Abelcet) that may be less nephrotoxic in those patients who cannot tolerate conventional amphotericin is available. Amphotericin B also causes infusion-related reactions including fever, chills, nausea, vomiting, headache, generalized malaise, hypotension, and arrhythmias. Major problems encountered less commonly than renal toxicity include anaphylaxis, generalized pain, bone marrow toxicity, rash, a variety of cardiac problems including cardiac arrest (choice B), liver failure (choice D), pulmonary edema (choice E), and convulsions (choice A). A 39-year-old woman presents to the emergency department after collapsing at a party. An interview with her boyfriend indicates that she complained of a severe headache prior to her collapse. He states that she has no significant past medical history and takes occasional vitamin supplements. Her blood pressure is 200/120 mm Hg, pulse is 37/min, and respirations are 5/min. The patient is unresponsive to commands or painful stimuli. There is moderate papilledema. The remainder of the examination is unremarkable. An electrocardiogram demonstrates normal sinus rhythm without T wave inversions or ST segment changes. Q1 Which of the following is the most likely diagnosis? / A. Anterior communicating artery aneurysm rupture / B. Atonic seizure / C. Cocaine induced myocardial infarction / D. Posterior inferior cerebellar artery aneurysm rupture / E. Vein of Galen malformation

Q2 Which of the following conditions would predispose this patient to having this condition?

/ A. Atherosclerosis / B. Diabetes / C. Hemophilia / D. Marfan syndrome / E. Protein C deficiency / F. Protein S deficiency Q3 A CT scan would most likely demonstrate blood in which of the following areas? / A. Fourth ventricle / B. Lateral ventricles / C. Subarachnoid space / D. Subdural space / E. Superior sagittal sinus / F. Third ventricle

Q4 Which of the following drugs could have precipitated this patient's condition? / A. Cocaine / B. Hashish / C. Lysergic acid diethylamide (LSD) / D. Morphine / E. Pindolol Q5 Which of the following is more likely to be present in patients with this condition than in normal persons? / A. Early AIzheimer disease / B. Fronto-temporal brain atrophy / C. Medullary thyroid carcinoma / D. Osteosarcoma / E. Renal cysts ____________________________________________________________________

Headache case 4 answers A1 The correct answer is A. This patient is presenting with a loss of consciousness, bradycardia, hypertension, and decreased respirations. While loss of consciousness has a wide differential diagnosis, the triad of bradycardia, hypertension, and decreased respirations is known as Cushing's triad, and is indicative of increased intracranial pressure. The finding of papilledema confirms that there is increased intracranial pressure. The differential diagnosis at this point is a spontaneous hemorrhage due to aneurysm rupture, trauma, vascular malformation rupture, or possibly a massive ischemic stroke. Of the choices given, anterior communicating artery aneurysm rupture is the most likely diagnosis. Aneurysms are outpouchings of the arteries of the Circle of Willis that occur most commonly at the anterior communicating artery, middle cerebral artery, or posterior communicating artery. They most commonly present with hemorrhage or headache. In this case, there is likely hemorrhage and increased intracranial pressure leading to secondary brain herniation. Treatment of this patient consists of lowering intracranial pressure and treating the aneurysm surgically. An atonic seizure (choice B) is a fainting spell in which the patient becomes hypotonic, but recovers over a short interval. There would be no signs of increased intracranial pressure. Myocardial infarction (choice C) from cocaine or other etiology would usually have electrocardiogram abnormalities and there would be no signs of increased intracranial pressure. Posterior inferior cerebellar artery aneurysm rupture (choice D) is a possibility but these aneurysms are rare, compared to anterior communicating artery aneurysms. Vein of Galen malformations (choice E) are a remnant of the fetal circulation that presents in children as a posterior fossa mass. Actual hemorrhage of these lesions is relatively rare. A2 The correct answer is D. Connective tissue diseases, such as Marfan syndrome, weaken blood vessel walls and predispose to aneurysms of any blood vessels in the body. Marfan syndrome is an autosomal dominant disorder that has been linked to the FBN1 gene on chromosome 15. FBN1 encodes the protein fibrillin, which is involved in the formation of elastic fibers found in connective tissue. Without the structural support provided by

fibrillin, many tissues are weakened, with severe consequences, e.g., aneurysm formation. Atherosclerosis (choice A) is not thought to be associated with intracranial aneurysms, which are believed to form from congenitally weak areas at the junctions of blood vessels. Aortic aneurysms are closely associated with atherosclerosis. Diabetes (choice B) is not thought to be associated with intracranial aneurysms. However, diabetes leads to an increased incidence of atherosclerosis, which may lead to aortic aneurysms. The hemophilias (choice C) are blood clotting disorders that do not predispose patients to aneurysms. These patients bleed profusely from even minor vessel trauma, however. Protein C deficiency (choice E) and protein S deficiency (choice F) are blood clotting disorders leading to thrombosis of arteries and veins. This does not predispose a patient to aneurysms, however. A3 The correct answer is C. The subarachnoid space consists of the space between the pia, which adhere to the brain, and the arachnoid membrane. The circle of Willis, including the anterior communicating artery, lies in the subarachnoid space. Subarachnoid hemorrhage is a common presenting symptom of ruptured intracranial aneurysms. Aneurysmal subarachnoid hemorrhage is usually within the basilar cisterns, where the circle of Willis lies, while posttraumatic subarachnoid hemorrhage is usually over the cerebral convexities. Hemorrhage into the epidural or subdural space is usually secondary to trauma. Epidural hematomas occur from injury to the middle meningeal artery and subsequent hematoma formation, and are usually associated with a fracture of the temporal bone. Intraventricular hemorrhage (choices A, B, and F) is a much less common presentation of a ruptured aneurysm. Usually there will be subarachnoid hemorrhage and intraventricular hemorrhage, rather than isolated intraventricular hemorrhage. Intraventricular hemorrhage often leads to ependymitis and hydrocephalus from dysregulation of the normal cerebrospinal fluid production and resorption physiology. Subdural hematomas (choice D) are usually secondary to trauma, not bleeding aneurysms. Subdural hematomas form from injury to the bridging veins between the venous sinuses and the cortical draining veins. Subdurals are common in elderly patients because they usually have some degree of brain atrophy and these bridging veins are stretched thin. There is normally blood present in the superior sagittal sinus (choice E), which drains the cortical veins from the top of the cerebrum.

A4 The correct answer is A. The key here is to find the drug that leads to hypertension, and thus is likely to cause an aneurysm to rupture. Cocaine leads to episodic hypertension due to its sympathomimetic effects. It may be snorted, smoked, or injected. Cocaine use is associated with cardiac arrhythmia, myocardial infarction, stroke, and cerebral or aortic aneurysm rupture. Although not a cause of intracranial aneurysm formation, it may lead to aneurysm rupture. Cocaine is used for its central effects on dopaminergic neurons, and the sympathomimetic effects described above are unwanted side effects. Hashish (choice B) and marijuana contain delta-9- tetrahydrocannabinol (THC), which is used for its effects on the central nervous system. Other physical effects include reddening of the eyes, dryness of the mouth and throat, moderate increase in the heart rate, tightness of the chest (if the drug is smoked), drowsiness, unsteadiness, and muscular incoordination. Hypertension is not a common effect of THC. Lysergic acid diethylamide (LSD) (choice C) is a psychotropic amide with many poorlyunderstood central nervous system effects. Significant hypertension does not generally occur with LSD. Morphine (choice D) is an opiate analgesic, and would tend to lower blood pressure, rather than increase it. Pindolol (choice E) is a nonselective beta-adrenergic receptor blocker. In addition, pindolol has partial agonist activity, with significantly greater agonist than antagonist effects at beta-2 receptors. It has negative inotropic and chronotropic effects and thus is used as an antihypertensive agent. It would help prevent hypertension. A5 The correct answer is E. Patients with adult polycystic kidney disease have a much higher incidence of berry aneurysms than the general population. Hypertension that may accompany the eventual renal failure can contribute to aneurysm rupture and subarachnoid hemorrhage. Early Alzheimer-like changes (choice A) are observed in patients with Down syndrome. Fronto-temporal brain atrophy (choice B) is seen in Pick disease. Medullary thyroid carcinoma (choice C) is seen with increased frequency in multiple endocrine neoplasia (MEN) IIa and IIb. Osteosarcoma (choice D) is more frequent in patients with familial

Anda mungkin juga menyukai